R REVIEW MUSCULO-KNEES, HIPs, FOREARM & WRIST

¡Supera tus tareas y exámenes ahora con Quizwiz!

A 45-year-old woman presents to her primary care provider complaining of pain and paresthesias in the first three digits of her left hand. The symptoms worsen at night. She has a history of hypothyroidism. Which of the following tests is most likely to be positive in this patient? A Finkelstein test BMcMurray test CPhalen test DThompson test

Correct Answer ( C ) Explanation: Phalen's test can be used to evaluate for carpal tunnel syndrome. Carpal tunnel syndrome (CTS) is a collection of signs and symptoms caused by compression of the median nerve within the carpal tunnel. CTS is the most common compressive focal mononeuropathy. Increased intra-carpal canal pressure is a key cause of clinical CTS. Risk factors for the development of CTS include obesity, female sex, pregnancy, diabetes, hypothyroidism, rheumatoid arthritis, aromatase inhibitor use, and occupational factors (e.g. repetitive hand and wrist use, vibrating tool use, sustained wrist or palm pressure). Patients with CTS classically present with pain or paresthesias in the median nerve distribution. Symptoms are often worse at night and provoked by activities that involve flexing and extending the wrist. Patients with severe cases may notice weakness or clumsiness when using the affected hand. The Phalen and Tinel tests are physical exam tests used to aid in the diagnosis. The Phalen test involves flexing the wrist and pressing the posterior aspect of the hands together. The Tinel test is performed by tapping the wrist over the median nerve. The diagnosis of CTS is a clinical diagnosis. Nerve conduction studies and needle electromyography can be used to exclude other conditions and determine the severity of the median nerve injury. Mild to moderate disease is initially managed with nonsurgical approaches such as splinting, oral and injection glucocorticoids, and physical therapy. Surgical decompression is often recommended in patients who present with severe disease. The majority of women who develop CTS during pregnancy will improve after delivery. The Finkelstein test (A) is used to diagnose de Quervain tenosynovitis. De Quervain tenosynovitis presents with pain over the radial side of the wrist and thumb and decreased grip strength. The McMurray test (B) is used to evaluate for meniscus injuries. Patients with meniscal injuries may complain of knee pain and a sensation of locking or catching when flexing and extending the knee. The Thompson test (D) is used to evaluate for Achilles tendon rupture. Achilles tendon tears present with acute heal pain and an inability to push off with the foot. One Step Further Question: What are the superior and inferior boundaries of the carpal tunnel? Reveal Answer: Rapid Review Carpal Tunnel Syndrome Patient with a history of extensive wrist usage such as typing Complaining of pain and numbness in the first, second, and third digits, especially at night PE will show Phalen's sign: reproduction of symptoms with wrist hyperflexion, Tinel's sign: reproduction of symptoms with percussion over carpal tunnel Most commonly caused by median nerve compression Treatment is NSAIDs, volar splint in neutral position

What is intersection syndrome?

An overuse tendinopathy manifesting with pain to the radial side of the wrist 4-8 cm proximal to the site of de Quervain's disease.

Which of the following is the most common type of shoulder dislocation? AAnterior BInferior CPosterior DSuperioR

A Anterior Shoulder dislocations are very common, due in part to the inherent instability of the shoulder joint. Anterior dislocations are most common and make up 95-97% of all shoulder dislocations. The mechanism of injury is generally due to a blow to the arm that is abducted, externally rotated, and extended. Sports activities such as blocking a basketball shot, motor vehicle accidents and a fall on an outstretched arm can all lead to anterior shoulder dislocation. Treatment involves reduction of the shoulder, immobilization and referral to an orthopedic specialist within one week of the injury.

A 75-year-old man with a past medical history of lung cancer, type II diabetes mellitus, and hypertension, presents to the emergency department with a chief complaint of three days of low back pain with radiation into both lower extremities, saddle distribution paresthesia, and bowel and bladder incontinence. His last A1C was 7.9. What disorder should be highest on the differential diagnosis? ACauda equina syndrome BDiabetic neuropathy CIntramedullary metastases DSpinal stenosis

A Cauda equina syndrome Cauda equina syndrome occurs when there is compression of the lumbosacral nerve roots below the level of the conus medullaris, or the most distal portion of the spinal cord. Causes of cauda equina syndrome include tumors or metastases, spinal infection, or herniation of a lumbar disc causing narrowing of the spinal canal. Compression of these nerve roots results in neuromuscular and urogenital symptoms such as low back pain, peripheral neuropathy, saddle sensory paresthesia, and bowel and bladder dysfunction. This is a medical emergency because if the nerve roots remain compressed, lasting damage will ensue. Treatment is targeted toward the cause of the compression and involves surgical decompression. Diabetic neuropathy (B) is a complication of uncontrolled diabetes mellitus. Initial symptoms include loss of sensation throughout the distal toes and feet with eventual extension upward in a "stocking-glove" distribution. Patients may complain of peripheral numbness and tingling as well. Bowel and bladder incontinence, however, is not typically a symptom of diabetic neuropathy and therefore in the case above this disorder should be placed lower on the differential diagnosis. Intramedullary metastases (C) is a very rare finding, but a history of lung cancer does account for roughly 50% of the reported cases. Symptoms of intramedullary metastases include low back pain, motor weakness, and possible bowel or bladder dysfunction. In comparison to cauda equina syndrome, intramedullary metastases typically presents with unilateral weakness, but cauda equina syndrome presents with bilateral symptoms. Spinal stenosis (D) is the narrowing of the spinal canal, which results in symptoms of neurogenic claudication, or pain that is exacerbated by standing or walking and relieved with sitting or lying down. While spinal stenosis causes back pain and can cause sensory loss and weakness, it typically does not cause saddle distribution paresthesia nor bowel or bladder incontinence. If patients complain of bowel or bladder incontinence in the setting of low back pain, your suspicion for cauda equina syndrome should be high.

A 44-year-old man presents with insidious and atraumatic proximal lower leg pain. It is more tender during rest than exercise. It commonly occurs at night, and doesn't seem to improve with oral analgesics. Knee testing is negative for meniscal or ligamentous injury. A screening radiograph reveals no fracture, but does show a non-speckled calcified lesion within the proximal tibia. A bone scan shows no spine or pelvic lytic lesions. Which of the following is the most likely diagnosis? AChondrosarcoma BEnchondroma CMultiple myeloma DOsteoid osteoma

A Chondrosarcoma Primary malignant bone tumors are rare and benign bone tumors are common. However, metastatic bone tumors are very common, and skeletal metastasis must be strongly considered in any patient over 40 years of age with any bony lesion. The most common bone tumors in those over 40 years of age are metastases, chondrosarcoma, fibrosarcoma and multiple myeloma. Constant deep achy pain, especially in the nighttime hours and not relieved by rest, is the main symptom of bone malignancy. Large benign tumors may present in a similar fashion. Mass is a less common presenting complaint, but may suggest a benign condition especially if it is nontender. Chondrosarcoma and fibrosarcoma are common bone malignancies in adult metaphyseal bones. Osteosarcoma also has a metaphyseal predilection, however, it occurs with lytic and blastic lesions in the second decade, with chondrosarcoma having calcified lesions and fibrosarcoma having lytic lesions. Radiographic examination is necessary. CT is best used in evaluating benign bone lesions. MRI is better for evaluating malignant bone lesions. Bone scans are used to detect other sites of skeletal lesions. A chest radiograph and CT are usually ordered if a malignant bone tumor is suspected. Routine laboratory testing is of limited use, however, in those over 40 years of age, consider urinalysis, urine and serum protein electrophoresis and PSA testing. Benign tumors may be observed or excised, while malignant tumors are excised and treated with chemotherapy and radiotherapy.

A 27-year-old man presents after a fall. He struck his leg on a table and sustained a fracture of the fibular neck. Which of the following nerves is commonly injured in this type of injury? A Common peroneal B Femoral C Posterior tibial D Saphenous

A Common peroneal Correct Answer ( A ) Explanation: Fractures of the fibula usually occur as a result of direct blunt trauma or varus stress to the knee. Isolated fractures of the fibula are frequently inconsequential because the fibula is almost completely a non-weightbearing bone. The exception to this is the significant injury of a Maisonneuve fracture when the proximal fibular fracture is associated with disruption of the medial portion of the ankle joint. The common peroneal nerve passes around the neck of the fibula and its function must be evaluated in the setting of a proximal fibula fracture. The common peroneal nerve is a branch of the sciatic nerve and divides into the deep and superficial branches. These nerves provide sensation to the anterior and lateral portion of the leg and to the top of the feet. The motor innervation is for dorsiflexion of the foot. As a result, a patient with a peroneal nerve injury will develop a foot drop. The femoral nerve (B) is the largest branch of the lumbar plexus arising from L2-L4. It emerges from the psoas muscle in the upper thigh and does not anatomically cross near the proximal fibula. The posterior tibial nerve (C) is the larger branch of the sciatic nerve (along with the peroneal nerve). In the popliteal fossa, the posterior tibial nerve travels with the popliteal vein and artery and gives off branches innervating the gastrocnemius, popliteus and soleus muscles. The saphenous nerve (D) is a cutaneous branch of the femoral nerve. It travels along the medial aspect of the leg along the tibia and is not at risk of injury from a fibular fracture. One Step Further Question: What vascular structure is at risk of injury with a proximal fibular fracture? Answer: Anterior tibial artery. Rapid Review Lower Extremity Nerve Deficits Superior gluteal: Trendelenburg sign/gait (lesion is contralateral to the dropped side) Inferior gluteal: ↓ hip extension Obturator: ↓ thigh sensation (medial), adduction Femoral: ↓ thigh flexion, leg extension Common peroneal: ↓ foot eversion/dorsiflexion Tibial: ↓ foot inversion/plantarflexion

A 42-year-old man complains of difficulty washing his face and combing his hair with his right hand. On examination a nodule, band, and slight contracture are noted in the palm proximal to the fourth finger. This patient's symptoms are associated with which one of the following? A Diabetes mellitus B Hyperparathyroidism C Hyperthyroidism D Hypothyroidism

A Diabetes mellitus Correct Answer ( A ) Explanation: The patient has Dupuytren's disease (Dupuytren's contracture), which is most common in men over 40 years of age. It is a progressive condition that causes the fibrous fascia of the palmar surface to shorten and thicken. Patients present with one or more small, pitted nodules on the palm, which slowly progress to contracture of the fingers. Patients usually have difficulty with tasks such as face washing, hair combing, and putting their hands in their pockets. Grade 1 disease initially can be managed expectantly, but injecting the nodule with a steroid injection can be helpful. Surgery is recommended if function is impaired, contracture is progressing, or severe deformity is disabling. There is a strong association between diabetes and Dupuytren's disease. Studies have found a 3 to 33 percent prevalence of Dupuytren's in patients with diabetes; however, these patients tend to have a mild form of Dupuytren's with slow progression. Greater alcohol intake per week is associated with increased risk of Dupuytren's disease, but most patients with the disease are not alcoholics. Smoking also increases the risk of the disease. Although the reason for this association is unclear, microvascular changes in smokers may play a role. Hyperparathyroidism (B) is a disease caused by a disregulation in calcium. If symptoms are present, they are attributable to hypercalcemia and may include weakness, easy fatigability, anorexia, or anxiety. Untreated hypothyroidism (D) can contribute to hypertension, dyslipidemia, infertility, cognitive impairment, and neuromuscular dysfunction. Hypothyroidism does not have any correlation with dupuytren's disease. Hyperthyroidism (C) causes anxiety, hyperactivity, heat intolerance, muscle weakness, hand tremors and much more. However it does not cause dupuytren's disease. One Step Further Question: Which pathological disorder is associated with pretibial myxedema? Answer: Hyperthyroidism, specifically Grave's disease

Lateral epicondylitis involves inflammation at the attachment of which of the following muscles? A Extensor carpi radialis brevis B Flexor carpi radialis C Pronator teres D Triceps

A Extensor carpi radialis brevis Correct Answer ( A ) Explanation: Lateral epicondylitis, commonly referred to as tennis elbow, is an inflammatory condition involving the lateral epicondyle of the humerus. The lateral epicondyle is the site of origin for the wrist and digit extensors and the forearm supinators. Muscles that originate at the lateral epicondyle include the extensor carpi radialis brevis, extensor digitorum, extensor digiti minimi, and the extensor carpi ulnaris. Lateral epicondylitis is often the result of repetitive supination of the forearm. Patients present with focal tenderness over the lateral epicondyle as well as proximal wrist extensor muscle mass pain with resisted wrist extension with the elbow in full extension, and pain with passive wrist flexion with the elbow in full extension. Treatment includes nonsteroidal anti-inflammatory medications and activity modification. A counterforce brace placed distal to the elbow and physical therapy is recommended as well. The flexor carpi radialis (B) and the pronator teres (C) both originate at the medial epicondyle of the humerus. Medial epicondylitis presents with focal tenderness over the medial epicondyle and proximal wrist flexor muscle mass pain with resisted wrist flexion with the elbow in full extension and pain with passive terminal wrist extension with the elbow in full extension. The triceps (D) tendon inserts on the olecranon. Triceps tendonitis will present with pain and tenderness of the posterior elbow which is worse with extension of the forearm. One Step Further Question: What elbow ligament is most commonly injured in pitchers? Answer: The ulnar (medial) collateral ligament

In what time frame should a surgical fasciotomy be performed in order to be most effective?

Answer: Six hours from onset of injury.

A factory worker sustains a crush injury to his lower left leg. He has 9/10 sharp pain, distal pallor, and paresthesias. The skin is intact but swollen. Minimal passive plantar flexion reproduces significant pain. Which of the following measurements are necessary to confirm a diagnosis? AAnkle-brachial index and wedge pressure BDiastolic blood pressure and intracompartmental pressure CMean arterial pressure and jugular venous distension DStroke volume and heart rate

B Extremity compartments are bound by fascial planes and contain muscles, nerves, and arteries. Acute compartment syndrome occurs when the vascular supply is unable to properly perfuse these structures. Cases are typically acute and traumatic, especially when long bones such as the femur or tibia fracture, or when there is a crush injury. Pathologically, injury causes intracompartmental pressure to build, which decreases venous outflow and backs-up arterial inflow, which ultimately leads to ischemic necrosis in as little as four hours. Pain out of proportion to injury, distal paresthesias, distal paresis or paralysis, and distal pallor are common presenting symptoms. A key physical finding is intolerable passive stretching of muscles around an injury site. Since necrosis is imminent if left untreated, suspected compartment syndrome needs immediate diagnosis and treatment. Compartment syndrome is confirmed when the diastolic blood pressure minus the intracompartmental pressure is ≤ 30 mm Hg or absolute compartment pressure > 30-40 mm Hg. Once confirmed, surgical fasciotomy of the compartment is essential for a favorable prognosis. The wound is left open. Delayed closure or skin grafting is performed after edema has resolved.

A 44-year-old man presents to the emergency department with back pain. On exam, the man appears uncomfortable and is tender to palpation over the right paraspinal muscles. Which of the following characteristics would most likely necessitate further evaluation? A) Back pain lasting more than ten days B) History of chronic corticosteroid use C) History of peptic ulcer disease D) Pain radiating down the posterior thigh during straight leg test

B History of chronic corticosteroid use A history of chronic corticosteroid use is a risk factor for vertebral compression fractures and would most likely require further evaluation. Low back pain a very common complaint seen in primary care setting and in the emergency department. Risk factors for back pain include smoking, obesity, increasing age, female gender, sedentary work, and strenuous physical work. The majority of low back pain cases are due to musculoskeletal causes. Rare, but serious systemic etiologies of back pain include cauda equina syndrome, metastatic cancer, epidural abscess, vertebral compression fracture, and vertebral osteomyelitis. In order to help rule out serious etiologies, patients who present with low back pain should be questioned about constitutional symptoms, history of malignancy, neurologic symptoms, recent bacterial infections, history of injection drugs, history of corticosteroid use, and recent history of back procedures. Physical exam should include inspection of the back, palpation of the spine, straight leg raise, and neurologic exam. Laboratory studies are not typically required for the majority of patients who present with low back pain. If underlying infection or malignancy is suspected, the erythrocyte sedimentation rate (ESR) or C-reactive protein (CRP) may be useful. Routine plain radiographs are not recommended for patients with back pain lasting less than four weeks. Plain radiographs are indicated in patients with risk factors for compression fractures. A magnetic resonance image (MRI) is indicated for patients with any neurologic deficit and in patients with a high clinical suspicion for epidural abscess. First-line pharmacologic treatment for subacute and chronic low back pain is acetaminophen or nonsteroidal anti-inflammatory drugs (NSAIDs). Second line treatment options include short-term opioids, short-term muscle relaxants, and antidepressants. Physical therapy is also beneficial in patients with subacute and chronic low back pain. Spinal fusion is the most common surgery for chronic nonspecific back pain with degenerative changes.

A 73-year-old woman presents with right knee pain for the past several months that has been worse over the last three days. It is worse with walking and better with rest. She has not tried anything to alleviate the pain. Physical examination reveals bony enlargement of the knee and joint line tenderness. An X-ray is ordered and shows narrowed joint spaces, and subarticular reactive sclerosis. She denies any injury or previous surgery on the knee. Which of the following is the best initial pharmacologic treatment? AGlucosamine plus Chondroitin BIbuprofen CPrednisone DTramadol

B Ibuprofen Ibuprofen has the lowest side-effect profile with the most significant pain relief and is the initial analgesic of choice for osteoarthritis. In mild disease, topical NSAIDs are recommended prior to the initiation of oral NSAID use. Acetaminophen can also be used when NSAID use in contraindicated. With the use of tylenol it is important to consider the safety concerns associated with overuse and an increased awareness of its negligible and non-clinically significant effects on pain. History will generally reveal mechanical pain and physical exam may be relatively benign or may reveal crepitus, joint line tenderness, and bony enlargement. It is the most common joint diseases affecting older adults and can lead to disability. For many patients, adequate monotherapy is enough to control symptoms of pain. Other nonpharmacologic treatments include exercise, weight loss (if necessary), and avoidance of aggravating activities.

A 19-year-old man who is employed in construction presents with pain to his thumb and wrist for 4 months. He states that the pain worsens while he is working. Examination reveals pain along the radial surface of the wrist with forced ulnar adduction. What management is indicated? AMRI of the wrist BNSAIDs and thumb spica splint CSurgical release of the dorsal extensor compartment DWrist X-ray with ulnar deviation view

B NSAIDs and thumb spica splint The patient presents with de Quervain's disease, or tenosynovitis, and should be managed conservatively with splinting and NSAIDs. De Quervain's disease is a repetitive strain injury that is often work related. In this disorder, the abductor pollicus longus (APL) and extensor pollicus brevis (EPB) tendons found in the dorsal extensor compartment of the wrist are inflamed and enlarged. Patients report pain on the radial surface of the wrist, which is increased with movement of the thumb or ulnar deviation at the wrist. Patients may have decreased grip strength as well. Finkelstein's test is considered to be pathognomonic for de Quervain's disease. It involves ulnar deviation of the wrist with the thumb in full flexion. This movement creates severe pain over the styloid of the radius. First line treatment is with rest, splinting (thumb spica splint) and NSAIDs. MRI of the wrist (A) is not necessary in the diagnosis of de Quervain's disease but is useful if a scaphoid fracture is suspected. Surgical release of the dorsal extensor compartment (C) is indicated for patients with refractory symptoms. A wrist X-ray with ulnar deviation (D) can reveal a small scaphoid fracture in trauma to the wrist.

A 55-year-old man presents to your office with gradual onset atraumatic low back pain, leg pain, and numbness. The pain is most often bilateral, involving the buttocks and thighs and spreads distally towards the feet. The leg pain begins and worsens with walking and standing and is relieved by sitting or lying down with hips and knees drawn up in a sitting posture. Which of the following is the most likely diagnosis? AAnkylosing spondylitis BScoliosis CSpinal stenosis DSpondylolisthesis

C Spinal stenosis is the narrowing of the spinal canal and neural foramina. The more common acquired spinal stenosis is caused by degenerative changes in the intervertebral disks, ligaments, and facet joints surrounding the lumbar canal. These degenerative changes can be caused by disk or joint disease, back surgery, and repetitive trauma. Stenosis initially becomes symptomatic at 40 to 50 years and older. Congenital narrowing of the spinal canal causes symptoms earlier in life and is uncommon. Spinal stenosis usually occurs at cervical and lumbar segments. Patients with cervical stenosis present with radiating arm pain, numbness, paresthesia, and motor weakness. The common symptoms with lumbar stenosis are gradual onset low back pain, leg pain, and numbness. The pain is most often bilateral, involving the buttocks and thighs and spreading distally toward the feet. The classic presentation is radiating leg pain (burning or cramping) that begins or worsens with walking and standing and is relieved by sitting or lying down with hips and knees drawn up in a sitting posture (neurogenic claudication). Bending forward diminishes pain. Physical examination is frequently normal but may include loss of lumbar lordosis, impairment of spinal mobility, asymmetric knee or ankle reflexes, and muscle weakness. Results of straight-leg raising are characteristically negative. Advanced imaging studies are obtained to establish and confirm the diagnosis of spinal stenosis when surgery is considered. Plain spine radiographs often reveal degenerative changes. MRI is currently the preferred modality, followed by CT scan.

Which of the following stable patients with back pain should have an emergent MRI performed? A23-year-old man with a positive straight leg raise and otherwise normal neurologic examination B30-year-old woman with back pain and dysuriaYour Answer C34-year-old man with a history of intravenous drug abuse with back pain and constipation D45-year-old woman with back pain after a car accident with no midline tenderness and normal neurologic examination

C 34-year-old man with a history of intravenous drug abuse with back pain and constipation The majority of patients who present with back pain do not require emergent imaging. However, those who may have a serious cause of their back pain including cauda equina syndrome, epidural abscess, vertebral osteomyelitis and other causes of cord impingement should be considered for emergent imaging of the spine. Patients that present with back pain should be screened for "red flags" in their presentations that support one of these critical diagnoses. These red flags include age (both extremes), history of cancer (especially those known to metastasize to bone), fever, weakness, saddle anesthesia, change in bowel or bladder function (incontinence or retention/constipation), trauma, history of immunocompromise (including chronic steroid use) and intravenous drug abuse. Patients with these components to their presentations have a higher risk of dangerous diagnoses and should be considered for early imaging with either CT (better for bony abnormalities) or MRI (better for spinal cord visualization). A positive straight leg raise (A) (pain radiating from the back into the leg below the knee) is suggestive of a protruding disk but can be worked up as an outpatient. Patients with symptoms of pyelonephritis (B) rarely require imaging unless there is a more complicated picture to their presentation (presence of kidney stone, sepsis, septic shock). Patients with trauma but no midline tenderness and a normal neurologic examination (D) do not require imaging.

A 38-year-old woman presents to your office with complaints of wrist pain and swelling. Physical exam findings include a flesh-colored, smooth, firm, rounded swelling on the dorsal aspect of her wrist. Which of the following is the most appropriate first step in management? A Closed rupture by force B Corticosteroid injection C Cyst aspiration D Referral for surgical intervention

C Cyst aspiration Correct Answer ( C ) Explanation: Ganglion cysts are benign lesions and are one of the most common soft tissue swellings of the wrist and hand. Ganglion cysts are thought to be caused by a degeneration of the mucoid connective tissue, although a definitive etiology has yet to be established. They are most commonly seen at the dorsum of the wrist at the scapholunate joint, but may occur at any joint or tendon sheath. Patients may present with an obvious swelling or may have joint pain as the primary complaint without an obvious etiology. Initial treatment is with nonsurgical measures including observation or needle aspiration. Many patients experience spontaneous resolution of the ganglion cyst without intervention and recurrence is common regardless of intervention used. Historically, people believed that ganglion cysts could be cured by using force to cause a closed rupture (A), hence the expression "bashing it with a bible". This is not a recommended modality to treat ganglion cysts. Corticosteroid injection (B) should be avoided as it may cause thinning of the skin overlying the cyst. Referral for surgical intervention (D) may be appropriate after conservative measures such as observation and needle aspiration fail. One Step Further Question: What physical exam technique helps to differentiate ganglion cysts from other nodular lesions of the hand and wrist? Answer: Transillumination

An 18-year-old man presents with pain in his index finger. Which of the following is suggestive of flexor tenosynovitis? ADelayed capillary refill BHolding the finger fully extended CPain on passive extension DSwelling localized to the volar aspect of the finger

C Pain on passive extension Flexor tenosynovitis is an infection of the flexor tendon sheath of the finger. Most commonly, penetrating trauma to the actual sheath causes this infection although direct spread from other areas of the hand may occur. Staph aureus and streptococci are the most common causative organisms. Kanavel's signs are the four cardinal signs of acute flexor tenosynovitis include: 1) tenderness along the flexor tendon, 2) symmetric swelling of the finger ("sausage finger"), 3) pain on passive extension and 4) flexed posture of the finger. These patients need antibiotic therapy and admission to the hospital for possible operative drainage.

An 8-year-old boy was playing in his father's garden-shed and stepped on a nail. At that time, his father cleansed and dressed the wound appropriately. However, seven days later, the boy presents with difficulty walking. Examination shows a lateral sole puncture wound that is associated with surrounding warmth, erythema and edema. He also has an antalgic gait. Distal neurovascular examination is normal. An AP radiograph shows a round cuboid lucency. Which of the following is the most appropriate intervention at this time? ABone scan BOral antibiotics CSurgical debridement DTopical antibiotic ointment

C Surgical debridement In children, acute osteomyelitis commonly occurs with hematogenous spread or direct contamination, as in stepping on a nail. Pain, edema, erythema, warmth and generalized malaise are common symptoms of acute osteomyelitis. It is also important to consider cellulitis and septic arthritis in any case of suspected osteomyelitis. Laboratory investigation includes CBC, CRP and ESR, all of which may be normal early in the disease course. Blood cultures, which reveal the causative organism in up to 50% of cases, should also be obtained. Radiographs are also recommended. Early films may only show soft tissue swelling, but are useful in ruling-out fracture or neoplasm. Follow-up radiographs around a week after the injury usually shows periosteal elevation and bone lucency, two signs of bony destruction and significant pathology. In addition to blood cultures, aspiration is recommended if one can identify involvement of a specific bone and neoplasm has been ruled-out. Intravenous antibiotics are started immediately. Surgical debridement, especially in the case of nail puncture wounds, is strongly recommended, as failure to completely eradicate infection with Pseudomonas aeruginosa, the most common organism in nail puncture wounds, can lead to chronic refractory osteomyelitis, additional joint destruction, limb-length discrepancies, limb dysfunction and pathological fracture. A bone scan (A) or MRI may be considered for unclear cases of osteomyelitis. Timely, aggressive debridement trumps bone scanning in any case of highly suspected osteomyelitis, as is evident in the above patient presentation. Intravenous, not oral or topical (B and D), antibiotics are recommended in suspected acute osteomyelitis.

A 15-year-old basketball player presents with left knee pain and swelling. He was running toward the basket when he pivoted around another player and felt a pop in his knee. He dropped to the ground and couldn't play the rest of the game. Physical exam reveals a knee effusion and a positive Lachman test. What is the most likely diagnosis? A Anterior cruciate ligament tear B Intra-articular fracture C Medial meniscal tear D Patellar dislocation

Correct Answer ( A ) Explanation: Anterior cruciate ligament (ACL) tears usually occur after a sudden deceleration and a rotational maneuver. Women tend to be at higher risk for these injuries. Clinical features include a history of a rotational injury, a pop reported by the patient, and an effusion developing after the injury. Physical exam will reveal an effusion, a positive anterior drawer test (lack of an end-point with forward translation of the tibia), a positive Lachman test (increased translation of the tibia when compared to the femur), and a positive pivot shift test (anterior subluxation of the tibia during extension). Plain radiographs may demonstrate an effusion. Magnetic resonance imaging (MRI) is the diagnostic modality of choice and can identify ACL tears and meniscal injuries accurately about 90-95 percent of the time. ACL tears usually require surgical repair. This is typically done approximately three weeks after the initial injury. Physical therapy is also important to full recovery. Most individuals regain full function of the knee after surgical repair. Acutely, the patient should be placed in a knee immobilizer, given crutches, and made non-weight bearing. They should be prescribed oral analgesic medications and referred to orthopedics. Intra-articular fracture (B) should be considered but is typically a result of traumatic blow. A medial meniscal tear (C) would have a positive McMurray test, and the patient will often complain of instability or locking of the knee. Patellar dislocation (D) is usually secondary to severe hyperextension or fall onto a bent knee. This is most commonly a lateral dislocation, which will be present as a visible deformity. One Step Further Question: What are the Pittsburgh knee rules? Answer: Obtain a knee X-ray for any of the following: recent trauma, unable to take four unaided steps, and age > 50 or < 12 years. Rapid Review Anterior Cruciate Ligament (ACL) Tear Patient with a history of quickly stop moving and change direction while running Complaining of pop and swelling Diagnosis is made by Lachman's Test (most sensitive) and Anterior Drawer Test

A 6-year-old boy presents with a limp for the last several months. He complains of pain in his groin that is worse at the end of the day. He denies trauma. On examination, he is afebrile and well appearing. He has pain with internal rotation and abduction of the right hip. There is no effusion, redness, or warmth over the hip joint. Which of the following is the most likely diagnosis? A Legg-Calvé-Perthes disease B Osgood- Schlatter disease C Septic arthritis of the hip D Slipped capital femoral syndrome

Correct Answer ( A ) Explanation: Legg-Calvé-Perthes (LCP) disease or idiopathic avascular necrosis of the proximal femoral epiphysis, most commonly occurs in male children between the ages of 4 and 10 years with a peak between 5 and 7 years of age. It often presents with limp that has an insidious or stuttering onset. The associated pain is generally mild, refers to the groin, anteriomedial aspect of the thigh or knee, is often relieved by rest, and generally worse at the end of the day. Early in the disease, patients have pain with and limited abduction and internal rotation of the hip due to synovitis and muscle spasm. As the disease progresses, the limited abduction is permanent and due to bony changes in the femoral head. They may have a positive Trendelenburg's sign, thigh, calf and buttock atrophy or limb-length discrepancy from the collapse of the femoral head. LCP is diagnosed on plain films with AP and frog-leg views, bone scan and MRI. ED treatment involves pain management and orthopedic referral, as medical or surgical treatment may be necessary based on clinical and radiographic findings. The goal is to prevent chronic deformity, improve range of motion, limit growth disturbance, and prevent chronic degenerative changes. Osgood-Schlatter disease (B) is inflammation of the patellar ligament at the tibial tuberosity that is most commonly seen in adolescent males. It manifests as a painful lump inferior to the knee. Acute septic arthritis (C) most commonly occurs in males younger than 4 years of age and presents with acute onset fever, irritability, pain, and refusal to bear weight or move the affected hip. Generally the area is warm, erythematous, and swollen. The hip is often held in flexion, abduction and external rotation. Slipped capital femoral syndrome (D) occurs with posterior and inferior slippage of the proximal femoral epiphysis on the metaphysis and is most common in overweight, adolescent males between 12 and 16 years of age. Patients with SCFE present with progressive limp and pain localized to the groin, thigh or knee. One Step Further Question: What is a positive Trendelenburg's sign? Answer: When standing, a patient is asked to lift one leg up at a time. When the patient stands on the affected side the pelvis droops on the unaffected side due to weakened gluteal muscles

A 12-year-old boy is brought to his pediatrician's office for right knee pain that started three months ago after starting basketball practices. On physical exam, the boy is tender to palpation over the anterior tibial tubercle. There is no erythema or warmth to the area. A lateral radiograph of the knee reveals elevation of the tibial tubercle away from the shaft. Based on the likely diagnosis, which of the following is the most appropriate management? A Acetaminophen and continued activity as tolerated B Activity avoidance until pain resolution C Knee immobilization and crutches D Referral to orthopedics for glucocorticoid injection

Correct Answer ( A ) Explanation: Osgood-Schlatter disease is best managed with acetaminophen and continued activity as tolerated. Osgood-Schlatter disease is a traction apophysitis of the proximal tibial tubercle at the point of patellar tendon insertion. Osgood-Schlatter disease is caused by repetitive quadriceps contraction, which results in excessive patellar tendon traction on the skeletally immature tibial tendon. Osgood-Schlatter disease is most commonly seen in 9-14 year old children who have undergone a rapid growth spurt. It is more common in children and adolescents who are participant in sports. Osgood-Schlatter disease is more common in athletes who participate in sports that involve running, cutting, and jumping. Anterior knee pain that gradually worsens over time is the most common presenting complaint. Physical exam findings typically include tenderness and soft tissue or bony prominence of the tibial tubercle. The pain can be exacerbated by having the patient extend the affected knee against resistance or squatting with the knee in full flexion. Erythema and warmth are not typical findings and warrant further evaluation for more serious conditions, such as osteomyelitis or malignancy. Radiographs are not necessary in patients with clinical findings characteristic of Osgood-Schlatter disease, but may be used to exclude other conditions. Radiographic findings findings may include anterior soft tissue swelling or tibial tubercle abnormalities. Because Osgood-Schlatter disease is usually benign and self-limited, conservative measures are the mainstay of treatment. Conservative treatment includes pain control, continuation of activity, and physical therapy. Non-steroidal anti-inflammatory drugs (NSAIDs), acetaminophen, and ice may help control the pain. Patients should continue to participate in activity as tolerated. Surgical treatment is reserved for patients with persistent pain despite conservative management. The course of Osgood-Schlatter disease usually lasts 6-18 months. Osgood-Schlatter disease rarely causes permanent impairment or disability. Activity avoidance until pain resolution (B) is unnecessary for Osgood-Schlatter disease. It is one of the few conditions where playing with pain is permitted. Inactivity can result in deconditioning, which increases the risk of recurrence and other injuries after resuming sports. Knee immobilization and crutches (C) are not indicated. Knee immobilization is contraindicated. Extended immobilization can lead to muscle atrophy and worsening of the ultimate outcome. Referral to an orthopedist for glucocorticoid injection (D) are not generally recommended due to case reports of subcutaneous atrophy. Several small studies have shown that injection of hyperosmolar dextrose aids pain relief for patients who have impaired ability to participate in sports for more than three months despite conservative management. One Step Further Question: What is a relative contraindication for surgery in patients with unresolved Osgood-Schlatter disease? Answer: Skeletal immaturity. Rapid Review Osgood-Schlatter Disease Patient will be a boy athlete 10 - 15 years old Complaining of knee pain while running PE will show tenderness over the tibial tubercle Treatment is ice, NSAIDs, quadriceps stretchin

A 66-year-old woman presents with right knee pain. X-ray reveals the above image. What is the most likely diagnosis? AOsteoarthritis BOsteomalacia COsteomyelitis DOsteoporosis

Correct Answer ( A ) Explanation: Osteoarthritis is a common malady that affects over 26.9 million Americans, and is one of the most common causes of long-term disability in the United States. There are varied risk factors for developing osteoarthritis including age, mechanical stress, and metabolic disorders such as ochronosis. The main finding for osteoarthritis is pain, which is generally worse with activity and occasionally accompanied with stiffness after periods of inactivity. Occasionally physical exam will show joint-line tenderness and intermittent joint effusion. Initial pharmacologic therapy consists of oral analgesics, such as acetaminophen or NSAIDs. Eventually, steroid injections and then joint replacement may become necessary. While a thorough history and physical is often enough to diagnosis osteoarthritis, X-rays provide a definitive diagnosis, and help distinguish osteoarthritis from other inflammatory joint diseases such as rheumatoid arthritis. Radiographs of a joint affected by osteoarthritis will show subchondral sclerosis, joint-space narrowing, subchondral cysts, and osteophytes. Osteomalacia (B) is caused by an inability to form hydroxyapatite crystals in osteoid, the unmineralized phase of bone. Radiographs will often show transverse translucent bands known as Looser zones or Milkman pseudofractures. Osteomyelitis (C) is an infection of bone that causes periosteal elevation and erosion on X-ray. Osteoporosis (D) is a disorder of bone and mineral metabolism that causes decreased bone strength. Radiographs are helpful to diagnosis acute fractures as a result of osteoporosis, but bone mineral densitometry, not X-ray, remains the diagnostic tool of choice for the disease. One Step Further Question: What is the difference between rickets and osteomalacia? Answer: Rickets occurs in children whose growth plates have not fused, while osteomalacia occurs in adults with closed epiphyses.

A 55-year-old man presents with new and insidious onset groin pain that began 5 months ago. His medical history includes sickle cell anemia. He currently abuses alcohol. Examination reveals painful hip rotation and decreased range of motion. There is no midline pelvic pain with resisted hip adduction. Pelvic radiographs reveal mild subchondral sclerosis about the femoral head, but no joint space narrowing or osteophytosis. Which of the following is the most likely diagnosis? A Avascular osteonecrosis B Legg-Calve-Perthes disease C Osteitis pubis D Osteoarthritis

Correct Answer ( A ) Explanation: Osteonecrosis of the femoral head is usually caused by a traumatic disruption of its blood supply or poor circulation due to other diseases. It most commonly occurs bilaterally and in those aged 40-60 years. Risk factors include a history of hip dislocation or femoral fracture, rheumatoid arthritis, systemic lupus erythematosus, Crohn's disease, chronic steroid or alcohol use, myeloproliferative disorders and sickle cell disease. Insidious onset of deep groin, hip or buttock pain predominates, however, acute presentations occur with acute collapse of the necrotic femoral head. Examination typically reveals antalgic gait and decreased and painful hip flexion, rotation and abduction. Radiographs early in the disease may be normal, or may reveal the earliest radiographical sign of this disease, subchondral femoral head sclerosis. MRI is necessary when the diagnosis is strongly suspected and the initial radiographs are normal or only positive for subchondral femoral head sclerosis. Pre-collapse treatment includes risk factor modification, physical therapy and pain management. Hip arthroplasty is typically reserved for function limiting head collapse. Secondary osteoarthritis is a common disease complication. Although Legg-Calve-Perthes disease (B) shares similar etiology and disease pathology with hip avascular osteonecrosis, its onset is in childhood, not adulthood. Osteitis pubis (C) is inflammation of the pubic symphysis which typically occurs in cumulative overuse of the hip adductors (as in certain athletes). Midline pelvic pain, with or without radiation to the groin or hip, that is worse with resisted hip adduction is a strong indicator of this condition. The classic radiographic findings of an osteoarthritic joint are subchondral sclerosis, cyst formation, joint space narrowing and osteophytes. The subchondral sclerosis of hip osteoarthritis (D) typically occurs in both the femoral head and acetabulum. One Step Further Question: What is the MRI finding which confirms a diagnosis of hip avascular necrosis? Reveal Answer: Rapid Review Avascular Necrosis Causes: corticosteroids (most common), alcohol Most common site: femoral head MRI

A 67-year-old man with a history of gout presents with atraumatic left knee pain. Physical examination reveals an effusion with overlying warmth and erythema. There is pain with passive range of motion. He reports a history of gout in this joint in the past. What is the appropriate next step? A Arthrocentesis of the knee B Indomethacin for acute gout C Measurement of serum uric acid D Radiograph of the knee

Correct Answer ( A ) Explanation: Septic arthritis is a bacterial or fungal infection of a joint typically spread hematogenously unless there is direct bacterial contamination. The synovium is highly vascular and lacks a basement membrane making it susceptible to bacterial seeding. Certain conditions predispose individuals to septic arthritis including diabetes, sickle cell disease, immunocompromise, alcoholism or pre-existing joint disease like rheumatoid arthritis or gout. Fever is present in less than half of cases of septic arthritis so with clinical suspicion an arthrocentesis is indicated. The knee is the most common joint affected and patients have pain (especially on passive range of motion) and decreased range of motion often accompanied by warmth, erythema and fever. This patient may have an acute gouty flare, but the clinician must exclude an infection. On joint fluid analysis, the white blood cell count of a septic joint is typically > 50,000. Indomethacin (B) is a non-steroidal anti-inflammatory agent commonly used in the treatment of acute gout. Gout is arthritis caused by deposition of monosodium urate monohydrate crystals in the joint space. Acute flares involve a monoarticular arthritis with a red, hot, swollen and tender joint. Acute episodes of gout result from overproduction or decreased secretion of uric acid. However, measurement of serum uric acid (C) does not correlate with the presence or absence of an acute flare. A radiograph of the knee (D) may show chronic degenerative changes associated with gout but will not help to differentiate gouty arthritis versus septic arthritis. One Step Further Question: Where is the most common site for gout? Answer: First MTP joint (podagra). Rapid Review Septic Arthritis Age <35: N. gonorrhea S. aureus most common overall Hematogenous spread Fever, pain, ↓ ROM Knee (most common) Arthrocentesis (WBC >50,000 with >75% PMNs) IV ABX, surgical washout

A 16-year-old girl presents to the clinic with complaints of left knee pain, instability and swelling after sustaining a twisting injury to the knee ten days ago during a soccer game. Which of the following physical exam findings would be most consistent with an anterior cruciate ligament tear? A Positive Lachman test B Positive McMurray test C Positive posterior drawer D Posterior sag sign

Correct Answer ( A ) Explanation: The Lachman test is performed with the patient supine. The knee is flexed to 20-30 degrees and an anterior force is placed on the tibia. The examiner looks for laxity or increased tibial translation compared to the contralateral knee. The Lachman test is described as grade 1 if there is 0 - 5 mm of anterior tibial translation, grade 2 if there is 6 - 10 mm of translation, or grade 3 if there is greater than 10 mm of translation. There is also a modifier for the presence or lack of an endpoint (i.e., A = good endpoint; B = no endpoint). Other physical exam tests that can be used to detect the presence of an anterior cruciate ligament (ACL) tear include the anterior drawer test and the pivot shift test. To perform the anterior drawer test, the knee is flexed to 90 degrees and an anterior force is placed on the tibia. Laxity or increased anterior tibial translation is a positive test. The pivot shift test is performed with the patient supine. The knee is extended and internally rotated. The examiner places a valgus stress on the knee and moves the knee from extension to flexion. The examiner will feel a clunk in the knee at approximately 20 - 30 degrees of flexion in an ACL-deficient patient as the tibia reduces from its subluxated position. The McMurray test (B) is used to evaluate for the presence of a meniscus tear. The examiner should place a thumb and index finger over the medial and lateral joint lines. The knee is then passively flexed and extended with first a varus and internal rotation force on the knee (to evaluate for a lateral meniscus tear) and then a valgus and external rotation force (to evaluate for a medial meniscus tear). A palpable pop or click with pain is a positive test. Both the posterior drawer test (C) and posterior sag sign (D) are used to assess the posterior cruciate ligament (PCL). For the posterior drawer test, the knee is flexed to 90 degrees and a posterior force is placed on the tibia. Laxity or posterior tibial translation is a positive test. To look for a posterior sag sign, the patient is examined in the supine position. The hip is flexed to 45 degrees and the knee is flexed to 90 degrees with the patient's foot on the exam table. Increased posterior translation of the tibia on the femur compared to the contralateral side, as a result of gravity, indicates a PCL injury. One Step Further Question: What are the indications for non-operative management of a complete ACL tear? Answer: Bracing and lifestyle modifications should be considered in adult patients with a complete tear if their daily activities do not involve jumping, cutting, or heavy manual labor.

Which of the following tests is the most accurate physical examination maneuver to identity an injury to the anterior cruciate ligament? A Lachman's test B McMurray's test C Thompson test D Valgus stress

Correct Answer ( A ) Explanation: The anterior cruciate ligament (ACL) is one of the four major ligaments of the knee (anterior cruciate, posterior cruciate, medial collateral, and lateral collateral). It provides anterior stability to the knee joint and is the most commonly injured ligament of the knee. The Lachman's test is the most accurate test for the ACL. In this test, the knee is flexed to 20 or 30 degrees and the thigh is stabilized. The tibia is then pulled anteriorly to look for abnormal excursion. McMurray's test (B) is used to identify meniscal tears. It is performed with the patient lying supine on the stretcher at which time the examiner flexes the foot and extends the knee while internally and externally rotating the tibia on the femur. A clicking indicates a positive test. The Thompson test (C) is performed to evaluate the integrity of the Achilles tendon. The patient's calf is squeezed which causes plantarflexion of the ankle joint. If flexion does not occur with the squeezing, the Achilles tendon is injured. Valgus stress (D) applied to the knee joint assesses the integrity of the medial aspect of the joint including the medial collateral ligament. One Step Further Question: What structures of the knee make up the unhappy triad? Answer: Anterior cruciate ligament (ACL), medial collateral ligament (MCL), and medial meniscus. Rapid Review Anterior Cruciate Ligament (ACL) Tear Patient with a history of quickly stop moving and change direction while running Complaining of pop and swelling Diagnosis is made by Lachman's Test (most sensitive) and Anterior Drawer Test

A 53-year-old man presents with a swollen, painful left knee. He denies trauma or fever. Examination reveals decreased range of motion of the left knee with swelling and warmth. What test is likely to be diagnostic? A Arthrocentesis and fluid analysis B Serum uric acid level C Serum white blood cell count D X-ray of the knee

Correct Answer ( A ) Explanation: The patient presents with atraumatic, monoarticular arthritis with warmth and swelling and must have an arthrocentesis to rule out a septic joint. Patients with septic arthritis will typically present with fever, joint pain, swelling and effusion in a single joint. Additionally, patients frequently have warmth of the joint and decreased range of motion. Involvement of multiple joints raises the possibility of meningococcal disease and sepsis. Most septic joints are caused by gram-positive bacteria (75-90%) with Staphylococcus aureus being the most common. Although there are a number of serum lab tests that are often requested in the workup of a possible septic joint (WBC, ESR, CRP) systematic reviews show little evidence that these tests can rule in or out the disease. The only way to obtain a definitive diagnosis is with arthrocentesis and fluid analysis. A synovial fluid WBC count is traditionally considered the most useful test. A WBC >50,000 cells is highly suggestive of a septic joint. Synovial lactate level >5.6 mmol/L has been found to be highly predictive of a septic joint, as well. Treatment of a septic arthritis involves antibiotics and orthopedic consultation for possible open drainage. A serum uric acid level (B) is helpful in the diagnosis of gouty arthritis but not in septic arthritis. Serum WBC count (C) is neither sensitive nor specific for this diagnosis. Radiographs (D) are often performed and may shows effusion or possibly osteomyelitis but are not diagnostic. One Step Further Question: How reliable are ESR and CRP in ruling out a septic joint? Answer: Not very. ESR is normal in up to 30% of patients and CRP is normal in up to 12% of patients

A 52-year-old man with a history of chronic alcoholism and hypertension presents to the Emergency Department with weakness of the right wrist that he first noticed when he woke up this morning. On examination, he is unable to extend the wrist against resistance. Which area of his hand is likely to have decreased sensation? A Dorsal first web space B Thenar eminence C Volar tip of the index finger D Volar tip of the pinky finger

Correct Answer ( A ) Explanation: The radial nerve, formed from C6-C8 nerve roots, courses down the medial upper arm before it wraps around the mid-humerus and continues more posteriorly. It is this area, the spiral groove, that is prone to compression injury and subsequent neuropraxia. Prolonged external compression can occur during sleep if the patient's head rests on the upper arm or the arm is hung over the edge of the bed or a bench. Sedation or intoxication increases the risk of this type of injury as the patient will not wake up despite paresthesias in the arm. Patients with a radial nerve palsy (also known as Saturday night palsy) will present with a wrist drop and be unable to extend the wrist against resistance. The fingers will be held in flexion at the MCP joint and the thumb is adducted. The radial nerve supplies sensation to the dorsum of the hand from the thumb to approximately the fourth metacarpal. The dorsal first web space is entirely within the radial nerve distribution and is the best location to confirm radial nerve involvement. Treatment includes splinting of the wrist and physical therapy. Prognosis is good for recovery of function in 3-4 months. Sensation of the thenar eminence (B) and the volar tip of the index finger (C) is supplied by the median nerve. The ulnar nerve supplies sensation to the volar tip of the pinky finger (D). One Step Further Question: What nerve is responsible for pincer grip between the thumb and index finger? Answer: Anterior interosseous branch of the median nerve.

A 19-year-old man with HIV presents with finger pain. Physical examination reveals the lesion shown above. What management is indicated? A Acyclovir B Cephalexin C Incision and drainage D Ritonovir

Correct Answer ( A ) Explanation: This patient has herpetic whitlow which is caused by the herpes virus. This is a self-limited condition. Immunocompetent individuals only require supportive care with analgesics and should cover the lesion to prevent autoinoculation. Immunocompromised individuals, as in the patient above, should be started on the antiviral, acyclovir. The lesion may be caused by HSV 1 or 2. Typically, the infection involves a single finger. It begins as pain or pruritus followed by the appearance of clear vesicles. These vesicles coalesce and can form an ulcer after 2 weeks. The diagnosis is made clinically but can be confirmed by a viral culture or Tzank smear from a scraping of an unroofed vesicle. The lesion resolves spontaneously in 3-4 weeks but recurrence occurs in 20% of patients. The main goals of treatment are to prevent oral inoculation or transmission and provide symptomatic relief. Cephalexin (B) is a first generation cephalosporin used in the treatment of bacterial skin and soft tissue infections. Incision and drainage (C) is contraindicated as it may lead to viral dissemination or bacterial superinfection. Ritonovir (D) is protease inhibitor used in the treatment of HIV infection. Although this patient is infected with HIV, administering ritonavir will not treat the herpetic whitlow. One Step Further Question: What is the role of topical acyclovir in herpetic whitlow? Reveal Answer: Rapid Review Herpetic Whitlow Clear painful vesicles Do not I&D Acyclovi

Which of the following structures is most at risk of direct damage with the condition shown above? anterior knee dislocation ACommon peroneal nerve BDeep femoral artery CPosterior tibial artery DSaphenous nerve

Correct Answer ( A ) Explanation: Tibiofemoral dislocation is a limb-threatening emergency caused by damage to multiple ligamentous structures in the knee joint. Tibiofemoral dislocations and their reduction threaten multiple neurovascular structures that run through and around the popliteal fossa. As such, examination should include a thorough neurovascular check, including an ankle-brachial index and detailed motor and sensory exams. Tibiofemoral dislocations, once recognized, should be immediately reduced to avoid permanent limb-threatening neurovascular damage. Serial neurovascular checks should follow, as damage to the popliteal artery, which runs through the popliteal space, and common peroneal (fibular) nerve, which winds around the fibular head, are common. The deep femoral branch of the femoral artery (B) runs along the posterior edge of the adductor longus muscle and extends only as far as the thigh. The posterior tibial artery (C) branches from the popliteal artery below the knee and provides blood to the posterior compartment of the leg and the plantar aspect of the foot. The saphenous nerve (D) is the cutaneous branch of the femoral nerve providing sensation to the skin of the patella. None of these structures are at risk of direct damage with a tibiofemoral dislocation. One Step Further Question: What is the motor function of the common peroneal nerve? Answer: Flexion of the leg at the knee through innervation of the short head of the biceps femoris muscle.

What is the classic description of ankylosing spondylitis?

Insidious onset of bilateral buttock pain (sacroiliitis), thoracolumbar pain (enthesitis) and lumbar morning stiffness that lessens with exercise most commonly seen in young men.

Which of the following is a common cause of non-traumatic avascular necrosis of the hip? A Cholecystitis B Chronic corticosteroid use C Chronic marijuana use D Iron deficiency anemia

Correct Answer ( B ) Explanation: Although a specific causative disorder is not identified in 20% of the cases, known atraumatic causes include chronic corticosteroid therapy, chronic alcoholism, hemoglobinopathy (e.g., sickle cell anemia), and chronic pancreatitis. When a patient has an increasingly painful hip, buttock, thigh, or knee and no history of recent trauma, avascular necrosis (AVN) of the femoral head should be considered. AVN is bilateral in 40 to 80% of patients. It is common in relatively young patients, the mean age at diagnosis is 38 years. AVN also is an emerging complication associated with human immunodeficiency virus (HIV) infection. It is unclear whether the virus itself or the treatments are the pathogenic agents. Chronic pancreatititis and alcoholism, not cholecystititis (A) or chronic marijuana use (C) have been linked to avascular necrosis. Sickle cell anemia and not iron deficiency anemia (D) can lead to non-traumatic avascular necrosis. One Step Further Question: Avascular necrosis is a complication of which traumatic hip injuries? Answer: Hip dislocation or femoral neck fracture.

Which of the following is true regarding the management of a subungual hematoma? A Antibiotics should be prescribed to patients after trephination of uncomplicated subungual hematomas B Complete nail removal may be necessary if the nail bed has been significantly damaged C Trephination is contraindicated when the subungual hematoma is associated with a distal phalanx fracture D Trephination is indicated only when the hematoma involves greater than 50% of the nail bed

Correct Answer ( B ) Explanation: Complete nail removal may be necessary if the damage to the nail bed is significant. A subungual hematoma is the result of a laceration to the nail bed. If the nail is stable and firmly adherent to the nail bed and the surrounding tissues are not significantly damaged, there is no reason to remove the nail and repair the underlying nailbed laceration. The nail itself will keep the nail bed laceration well approximated. Nail trephination in this scenario often results in a good cosmetic endpoint. Nail trephination is preformed under sterile technique with adherence to universal precautions. Local anesthesia is generally unnecessary for uncomplicated hematoma trephination. More complex injuries, including those in which nail removal is necessary, will require a digital block for patient comfort. Most commonly hot cautery (with a heated paperclip or disposable cautery device) is used. The device is inserted perpendicular to the nail in the center of the hematoma. A pop will be felt as the device passes through the nail and at this point the device should be removed to avoid damage to the nail bed. Blood will come out of the hole and the nail will return to a normal color. Gentle downward pressure applied by the patient will help blood escape. Occasionally large hematomas require more than one trephination site for adequate drainage. After the procedure, the area should be cleaned and dressed with a dry dressing. Splints are applied as indicated. The patient is instructed to keep the digit dry for 2 days. Alternative trephination techniques include the use of an 18-gauge needle twisted into the nail. Also a small 29-gauge needle can be inserted parallel and under the nail, past the hematoma site. This is useful for hard to drain second-fifth toe subungual hematomas. Subungual hematoma is often the result of slamming a finger in a door, striking a finger with an object or dropping an object onto a toe. This can result in a distal phalanx fracture as well. This is not a contraindication to nail trephination (C). The general rule is that all painful, acute (24-48 hours old) subungual hematomas should be drained irrespective of the size (D). Some sources cite that only hematomas greater than 50% should be drained, however smaller hematomas should be drained if the patient is experiencing significant discomfort. There is little risk of infection after trephination of an uncomplicated subungual hematoma therefore antibiotics are not prescribed (A). One Step Further Question: What is the differential diagnosis for non-traumatic subungual hematoma? Answer: Melanoma and Kaposi sarcoma

A mother is concerned with her newborn baby boy. She believes he doesn't move his right hip like he moves his left hip. You begin an examination with flexing both hips and knees. You come down to the level of the examination table and look at the heights of the patellae. You notice the right patellae sits much lower than the left. Since you are concerned with developmental dysplasia of the hip, you refer the mother to an orthopedist. In your consultation request, you state which of the following physical examination tests is positive? A Barlow B Galeazzi C Ortolani D Yergason

Correct Answer ( B ) Explanation: Developmental dysplasia of the hip (DDH) refers to hip instability caused by inadequate pressure of the femoral head against the acetabulum that leads to a shallow socket. This is associated with an increased risk of future hip dislocation and gait abnormalities. Risk factors include a positive family history, breech birth, and female gender. There are several examination tests used to screen for this condition. The Galeazzi (or Allis) test involves flexing the infant's hips and knees, then looking level at the knee heights. If unequal, DDH and congenital short femur may be present in the lower of the two knees. Early diagnosis of hip instability, and treatment with closed reduction, harnessing (Pavlik) or casting, usually results in a favorable prognosis. Later diagnosis may require surgery to correct gait abnormalities. The Barlow and Ortolani tests are used to test for infantile hip instability. The Barlow test (A) begins with hip flexion and adduction. Lateral force is then applied. The test is positive if this lateral force causes hip dislocation. The Ortolani (C) test begins with hip abduction. A dislocated hip will have less abduction than a properly-seated hip. As further abduction is attempted, a posteriorly-placed examiner's finger pushes anteriorly to lift the head back into the acetabulum, leading to a palpable "clunk" and improved passive abduction, which results in a positive test result. Yergason's test (D) is used in evaluating adolescent or adult biceps tendonitis, not congenital hip instability. One Step Further Question: Developmental dysplasia of the hip occurs with increased incidence if an infant also has which neck condition? Answer: Congenital torticollis ("wry-neck", most commonly caused by sternocleidomastoid fibrosis). Rapid Review Developmental Dysplasia of the Hip (DDH) ↓ Pressure of the femoral head against the acetabulum → shallow socket Galeazzi test: flexing the infant's hips and knees to compare knee heights Dx: < 4 mos: ultrasound Rx: Pavlik harness

A track-and-field athlete presents to orthopedic clinic with insidious onset anterior knee pain. Examination reveals tenderness over the lateral patella. Which of the following radiographic findings suggests bipartite patella over patellar fracture? A Narrowed patellofemoral joint space B Rounded patellar fragment C Sharply demarcated patellar fragment D Soft tissue swelling indicative of hemarthrosis

Correct Answer ( B ) Explanation: Failure of secondary ossification in one or two of the patellar poles is called bipartite or tripartite patellae. The missing bone is replaced by thick fibrous tissue. This is mainly an asymptomatic condition, however, it may be a cause of anterior knee pain. Symptom onset is insidious and may occur after repeated knee stretching seen in jumping sports, or symptoms can arise after frank trauma. In either case, disruption of the fibrous band is causative. During examination, tenderness over the unfused fragment is frequently encountered. Knee effusion or hemarthrosis is usually absent. Its presence, however, suggests patellar fracture. Furthermore, the radiograph can differentiate a disrupted bipartite patella from a patellar fracture, in that a bipartite fragment has rounded edges, and a patellar fracture has sharp fragment edges. Activity modification, which may include knee immobilization and crutches, is the main treatment. This gives the fibrous disruption time to heal and strengthen. A narrowed patellofemoral joint space (A) suggests a decrease in patellar cartilage, as in patellofemoral syndrome or chondromalacia patellae. It does not help differentiate bipartite patella from patellar fracture. A patellar fragment with sharp edges (C), or one associated with soft tissue swelling indicative of hemarthrosis (D), suggests the acute bony disruption of patellar fracture and not the fibrous disruption of bipartite patella. One Step Further Question: Which patellar pole is the most common site of osseous nonunion in bipartite patella? Answer: Superolateral. Rapid Review Patella Fracture MC: transverse Bipartite patella: normal variant, smooth cortical margins Assess extension ability Surgery indications: > 3 mm dislocation, extensor mechanism loss

You perform an arthrocentesis on a patient with knee pain. Synovial fluid analysis reveals a WBC 5,000 cells/µL with 70% PMNs. Which of the following is the most likely diagnosis? A Hemorrhagic effusion B Inflammatory arthritis C Noninflammatory arthritis D Septic arthritisYour Answer

Correct Answer ( B ) Explanation: The analysis of synovial fluid is essential for identifying crystalline and suppurative causes of acute arthritis. The WBC count dictates the inflammatory class of the fluid. However, an analysis is often difficult because there is significant overlap between entities. A very high fluid WBC or PMN pleocytosis indicates infection, but a modest elevation does not exclude it. Therefore, cell counts should be used as a guide rather than definitive diagnosis. WBCs between 2,000 and 75,000 are often categorized as inflammatory. However, a WBC over 50,000 has a likelihood ratio of 7.7 for septic arthritis. Hemorrhagic effusions (A) are caused by trauma to the knee or coagulopathy. The fluid is grossly bloody and may contain fat droplets (lipohemarthrosis). Noninflammatory arthritis (C) such as osteoarthritis will have <3,000 WBCs on arthrocentesis. The likelihood ratio for septic arthritis (D) increases as the joint WBC count rises. A WBC count <25,000 has a likelihood ratio of 0.32, >25,000 has a likelihood ratio of 2.9, >50,000 has a likelihood ratio of 7.7. One Step Further Question: What are two relative contraindications to performing an arthrocentesis? Answer: Overlying cellulitis and coagulopathy

A football player presents with acute anterior knee pain and significant knee effusion. He felt a "pop" while he was tackled. Which of the following physical examination tests is best used to diagnose a tear of the anterior cruciate ligament? A Anterior drawer test B Lachman test C McMurray test D Patellar grind test

Correct Answer ( B ) Explanation: The anterior cruciate ligament (ACL) is the primary stabilizer of the knee. It resists anterior translation of the tibia on the femur and is the most frequently injured major ligament of the knee. A tear results from deceleration, hyperextension or marked internal rotation of the tibia on the femur. The majority of ACL tears occur during sporting activities such as skiing, football, soccer, and basketball. Clinically, there is usually rapid development of hemarthroses, causing significant swelling. Almost half of individuals report feeling or hearing a pop, which is the most reliable factor. There is usually instability of the knee or a "wobbly" feeling. The Lachman test is the most sensitive test (approximately 98% sensitivity) showing increased anterior tibial displacement and a soft end point. MRI is rarely necessary to make a diagnosis. Rest, ice, crutches, immobilization and NSAIDs are common initial treatment options. Some competitive young patients may choose reconstruction as the best possible chance to return to successful play. The anterior drawer test (A) carries a 50% false-negative rate. As such, it is much less appropriate then the Lachman test for diagnosing ACL injury. The McMurray test (C) is used to diagnose meniscal, not ACL, injury. The patellar grind test (D) evaluates the articular surface of the patella, and is used in the evaluation of patellofemoral syndrome, not ACL tears. One Step Further Question: What is the most common cause of an acute hemarthroses after a sports-related knee injury? Answer: ACL tear. Rapid Review Anterior Cruciate Ligament (ACL) Tear Patient with a history of quickly stop moving and change direction while running Complaining of pop and swelling Diagnosis is made by Lachman's Test (most sensitive) and anterior drawer test

A 20-year old man presents after jamming his index finger during a basketball game. When he tries to straighten his finger, the distal tip remains flexed as shown above. Which of the following describes the pathophysiology of this injury? A Defect of the central slip causing volar migration of the lateral bands B Disruption of the extensor tendon mechanism at the distal interphalangeal phalangeal (DIP) joint C Disruption of the ulnar collateral ligament D Inflammation of the abductor pollicis longus and extensor pollicis brevis tendons

Correct Answer ( B ) Explanation: The patient has a mallet finger. A mallet finger is characterized by a closed disruption of the distal extensor apparatus, often occurring when there is sudden forceful flexion of an extended finger, such as when struck by a ball. Because the disrupted extensor tendon no longer connects with the distal phalanx, the DIP joint is held in flexion due to unopposed action of the flexor digitorum profundus. A mallet finger injury is the most common hand injury seen in athletes. On examination, the distal tip of the finger is flexed at the DIP joint. Pain, swelling, and tenderness may be present over the DIP joint. Passive extension is usually intact, but patients are unable to actively extend the distal phalanx. Radiographs may show an associated dorsal avulsion fracture. The treatment involves immobilizing the DIP joint in slight hyperextension for 6-8 weeks to allow tendon healing. The proximal interphalangeal (PIP) joint and metacarpophalangeal (MCP) joints should be allowed to move freely. A defect of the central slip causing volar migration of the lateral bands (A) is seen in a boutonniere deformity. Disruption of the ulnar collateral ligament (C) is the mechanism leading to gamekeeper's thumb. Inflammation of the abductor pollicis longus and extensor pollicis brevis tendons (D) is seen in de Quervain's tenosynovitis. One Step Further Question: What chronic finger deformity can result from an untreated mallet finger? Answer: Swan neck deformity. Rapid Review Mallet Finger Patient will be a basketball or volleyball player With a history of forced hyperflexion of the DIP Complaining of inability to extend DIP Treatment is volar splinting DIP in extension Comments: If untreated -> swan neck deformity

A 45-year-old man presents to the ED with pain and swelling of his right elbow. He does not recall any specific trauma. On exam, the patient has full passive range of motion with no pain. The elbow is slightly warm to touch. Which of the following is the most appropriate management for this patient? AAspirate fluid for cell count and culture BIce and NSAIDs CIncision and drainage DPosterior mold splint

Correct Answer ( B ) Explanation: The patient has olecranon bursitis. This is a painful inflammation of the bursa and is often secondary to local trauma, infection, or related systemic illness. Many times, patients cannot recall a specific traumatic event. Occupational risks related to excessive frictional forces include those who frequently place their elbows on a hard desk or a carpenter who spends long periods on his knees. Commonly involved sites include the olecranon, the greater trochanter of the femur, shoulder, and knee. Unless there is concomitant infection, treatment is conservative with ice and NSAIDs. There should also be temporary cessation of the activity that led to bursitis, or protective equipment should be worn, such as kneepads, for people doing prolonged work on their knees. Aspiration of the bursa (A) is an option if an infection is suspected. Septic bursitis typically presents with warmth and erythema over the joint and is sometimes difficult to differentiate from septic arthritis. Incision and drainage (C) are not recommended in bursitis unless there is associated development of an abscess. A posterior mold splint (D) is not indicated in bursitis. Some patients may experience comfort from a sling, but rarely is immobilization ever indicated. One Step Further Question: What is the most common type of bursitis? Answer: Subacromial (subdeltoid) bursitis. Rapid Review Olecranon Bursitis Swelling, pain ↓ Active ROM Full passive ROM NSAIDs

A 20-year-old man who is employed in construction presents with pain to his thumb and wrist for 4 months. He states that the pain worsens while he is working. Examination reveals pain along the radial surface of the wrist with forced ulnar adduction. What management is indicated? A MRI of the wrist B NSAIDs and thumb spica splint C Surgical release of the dorsal extensor compartment D Wrist X-ray with ulnar deviation view

Correct Answer ( B ) Explanation: The patient presents with de Quervain's disease, or tenosynovitis, and should be managed conservatively with splinting and NSAIDs. De Quervain's disease is a repetitive strain injury that is often work related. In this disorder, the abductor pollicus longus (APL) and extensor pollicus brevis (EPB) tendons found in the dorsal extensor compartment of the wrist are inflamed and enlarged. Patients report pain on the radial surface of the wrist, which is increased with movement of the thumb or ulnar deviation at the wrist. Patients may have decreased grip strength as well. Finkelstein's test is considered to be pathognomonic for de Quervain's disease. It involves ulnar deviation of the wrist with the thumb in full flexion. This movement creates severe pain over the styloid of the radius. First line treatment is with rest, splinting (thumb spica splint) and NSAIDs. MRI of the wrist (A) is not necessary in the diagnosis of de Quervain's disease but is useful if a scaphoid fracture is suspected. Surgical release of the dorsal extensor compartment (C) is indicated for patients with refractory symptoms. A wrist X-ray with ulnar deviation (D) can reveal a small scaphoid fracture in trauma to the wrist. One Step Further Question: What is intersection syndrome? Answer: An overuse tendinopathy manifesting with pain to the radial side of the wrist 4-8 cm proximal to the site of de Quervain's disease

A 53-year-old man presents with numbness to his right hand for three to four months. He states that he has numbness with waking up in the morning, which gets better when he shakes his hands. The patient indicates numbness to his first, second, and third digits on the right hand. Which of the following tests is most sensitive for this diagnosis? A Finkelstein's test B Median nerve compression test C Phalen's test D Tinel's sign

Correct Answer ( B ) Explanation: The patient presents with signs and symptoms consistent with carpal tunnel syndrome. Carpal tunnel syndrome is the most common entrapment neuropathy of the wrist and is caused by compression of the median nerve. Associated symptoms include numbness and weakness in the distribution of the median nerve. Patients typically present with numbness, pain and parasthesias of the first three digits that are worse at night and with activity but improve with "shaking their hands out." Median nerve compression testing (Durkan's test) consists of direct pressure application to the median nerve at the carpal tunnel. It is positive if this maneuver reproduces the symptoms and is 87% sensitive and 90% specific for the diagnosis. Conservative treatment with wrist splinting and cortisone injections often relieves symptoms. Referral for orthopedic management is indicated in patients with recurrent or refractory symptoms. Finkelstein's test (A) involves forced ulnar deviation of the wrist with the thumb adducted and is diagnostic of de Quervain's tenosynovitis. Phalen's (C) and Tinel's (D) test are classically taught in the diagnosis of carpal tunnel syndrome but suffer from low sensitivity (76% and 42-85% respectively). One Step Further Question: When is emergent orthopedic consultation required for carpal tunnel syndrome? Answer: Acute onset of symptoms over hours suggests the presence of acute median nerve compression from fracture or infection and requires emergent decompression

The scaphoid articulates with which of the following bones? A Hamate B Radius C Triquetrum D Ulna

Correct Answer ( B ) Explanation: The scaphoid bone is one of the eight carpal bones, which all together with the distal radius and ulna, make up the bones of the wrist. The carpal bones are arranged in two rows of four bones. The proximal row consists of the scaphoid, lunate, triquetrum, and pisiform, while the distal row consists of the trapezium, trapezoid, capitate, and hamate. The wrist has many complex articulations which allow for complex movements, including the radiocarpal joint, the midcarpal joint, and the distal radioulnar joint, which allow wrist flexion, extension, radial deviation (adduction), ulnar deviation (abduction), circumduction, pronation, and supination. The scaphoid and lunate bones articulate proximally with the radius to form the radiocarpal joint. The radius also articulates with the ulna at the distal radioulnar joint. The ulna does not directly articulate with the carpal bones, but rather has an interface with the triangular fibrocartilage complex, known as the articular disk, which binds together the distal ends of the radius, ulna, lunate, and triquetrum. Scaphoid fractures can be radiographically difficult to detect. In 15% of cases, radiographs taken immediately after injury fail to show a fracture line. Therefore, if a scaphoid fracture is suspected, the wrist should be immobilized in a thumb spica splint and follow-up for repeat clinical evaluation and radiographic imaging in 7-10 days.

A 44-year-old woman presents with pain to her second digit for 2 days. Physical examination reveals a small fluctuant area at the eponychium. What management is indicated? A Acyclovir x 7 days B Incision and drainage C Oral antibiotics and warm soaks D X-ray of the finger

Correct Answer ( B ) Explanation: This patient presents with a paronychia or a localized abscess of the nail fold that should be incised and drained. Patients will present with swelling and tenderness of the soft tissue at the lateral nail fold. Staphylococcus aureus is the most common causative agent in adults but in children, anaerobes may be more present (secondary to finger sucking). Management is with incision and drainage. The finger should be soaked in warm water and the edge of the skin at the nail should be gently lifted away from the nail using a scalpel blade. Subsequently, the cavity should be irrigated and packing gauze should be placed under the eponychium for 24 hours. If this were a herpetic whitlow, acyclovir (A) would be appropriate. Incision and drainage should be avoided for a herpetic whitlow as this can spread the infection. Oral antibiotics and warm soaks (C) are helpful if only cellulitis is present without abscess. X-ray of the finger (D) will not be helpful in the diagnosis or workup of a paronychia. One Step Further Question: In patients with chronic or unresponsive paronychia, what organisms are likely involved? Answer: Candida albicans and atypical mycobacteria. Rapid Review Paronychia Infection of lateral nail fold S. aureus Mild rx: ABX, warm soaks, Abscess rx: I&D

A 23-year-old woman, who is 6-weeks postpartum, presents with progressive right thumb and wrist pain for the past 3 weeks. The pain worsens with movement. Physical exam reveals tenderness over the right radial styloid. The pain is reproduced when the wrist is rotated ulnarly while the woman's fingers are folded over the thumb. Which of the following is the most likely diagnosis? A Carpal tunnel syndrome B De Quervain tendinopathy C Dupuytren contracture D Osteoarthritis of the trapeziometacarpal joint

Correct Answer ( B ) Explanation: This woman most likely has de Quervain tendinopathy. De Quervain tendinopathy is a tendinopathy of the abductor pollicis longus and extensor pollicis brevis tendons. De quervain tendinopathy is a frequent cause of atraumatic wrist pain in adults. De Quervain tendinopathy is most commonly seen in women ages 30-50 years. Risk factors include repetitive activities that involve abducting and extending the thumb. Postpartum mothers are thought to be at increased risk due to repeatedly lifting and holding the baby. Patients with de Quervain tendinopathy typically complain of pain on the radial side of the wrist. Patients may also notice decreased grip strength. Physical exam may reveal tenderness over the radial styloid. Finkelstein test can be used to reproduce the pain. The test involves having the patient wrap their fingers around the thumb and ulnarly deviating the wrist. Crepitus is rarely noted. Radiographs are normal for de Quervain tendinopathy. First-line treatment involves short-term nonsteroidal anti-inflammatory drugs (NSAIDs) and splinting. Glucocorticoid inject may be used for patients who do not improve with conservative therapy. De Quervain tendinopathy is typically self-limited in the majority of patients. Dupuytren contracture (B) is a fibrosing disorder that causes progressive thickening and shortening of the palmar fascia. Dupuytren contracture results in contractures of the metacarpophalangeal or proximal interphalangeal joints. The fourth and fifth fingers are most commonly affected. Patients may present with difficulty extending the affected finger and nodules may be palpated. Carpal tunnel syndrome (C) is characterized by numbness, paresthesias, and pain in the median nerve distribution. Carpal tunnel syndrome is caused by compression of the median nerve within the carpal tunnel. Physical exam may reveal decreased grip strength, numbness, or atrophy of the thenar eminence. Osteoarthritis of the trapeziometacarpal joint (D) would be unlikely in a 23-year-old woman. Patients with osteoarthritis typically present with deep, achy joint pain that improves within 30 minutes of movement. One Step Further Question: What is the innervation of the extensor pollicis brevis and abductor pollicis longus? Answer: Deep branch of the radial nerve. Rapid Review de Quervain Tenosynovitis Patient will be complaining of radial wrist pain PE will show pain on ulnar wrist deviation with thumb in fist (Finkelstein's test) Treatment is thumb spica splint, NSAIDs

A 12-year-old baseball player presents with knee pain. It occurs mainly during running the bases. Tenderness and edema are prominent about the right tibial tuberosity, and resisted knee extension is painful. The remainder of the knee examination is normal. Radiographic examination of this patient would most likely reveal which of the following abnormalities? A Bipartite patella B Heterotopic ossification C Increased density of the femoral condyles D Pars interarticularis defect

Correct Answer ( B ) Explanation: Tibial tuberosity osteochondritis is called Osgood-Schlatter disease (OSD). Repetitive quadricep contraction can lead to avulsions at the secondary ossification center between the tibial tuberosity and the patellar tendon. Incidence is greatest in adolescent boys who are active in sports. Symptoms include anterior proximal tibial pain, which is worse with running, jumping and prolonged sitting. Examination typically reveals unilateral tibial tuberosity tenderness and edema with restricted and tender kneeling. Lateral radiographs typically show spicules of heterotopic ossification anterior to the tibial tuberosity. Intermittent ice, NSAIDs and rest are common treatment options. Immobilization may be required for recalcitrant cases. Surgery is commonly unnecessary, and prognosis is typically good. Bipartite patella (A) may present with superolateral patellar pain, but not commonly with tibial tuberosity pain. Increased density of the femoral condyles (C) is not present in OSD. Increased density of the femoral head is common in Legg-Calve-Perthes disease, the childhood disorder of hip avascular osteonecrosis. A defect in the pars interarticularis (D), the bony junction of the superior and inferior articular processes of the vertebral posterior columns, can lead to spondylolisthesis and back pain, not knee pain. One Step Further Question: Which other condition mimics Osgood-Schlatter disease? Answer: Sinding-Larsen-Johansson syndrome, a similar pathology in the inferior patellar pole of boys aged 9-11 years.

An 18-year-old man is brought into the emergency room for evaluation after being the restrained passenger in a motor vehicle collision. He is alert and complains of severe right lower extremity pain. He was unable to bear weight on the extremity at the scene. On exam, there is presence of a transverse groove at the medial joint line The extremity is pale and cold. Popliteal pulses are not palpable in the right lower extremity and 2+ in the left lower extremity. Which of the following is the most appropriate next step in the management of this patient? A Emergent arteriography and vascular repair B Emergent open surgical reduction C Immediate fasciotomy D Immediate manual reduction

Correct Answer ( B ) Explanation: Tibiofemoral dislocation is a limb-threatening emergency caused by multiple ligamentous tears due to hyperextension, posterior force to the anterior tibia, or force to the femur or fibula. Motor vehicle collisions are the most common cause of tibiofemoral dislocations. Knee instability in multiple directions should raise suspicion for tibiofemoral dislocation, even if no gross abnormalities are present, as many will spontaneously reduce prior to presentation. Examination of a suspected tibiofemoral dislocation should include a thorough neurovascular check, including an ankle-brachial index and a motor and sensory exam. The presence of a transverse groove at the medial joint line is called a "dimple sign" and is an indication of a posterolateral knee dislocation, which cannot be manually reduced. Closed reduction is not possible because the medial femoral condyle is buttonholed through the medial capsule and the medial collateral ligament is invaginated into the joint. Posterolateral knee dislocations are a surgical emergency and need immediate open reduction. Arteriography (A) and possible repair will be necessary after reduction of this dislocation to evaluate for damage to the popliteal artery. This should be done after reduction of the knee as the tibiofemoral dislocation will continue to damage the artery until it is reduced. Fasciotomy (C) is the intervention of choice for suspected compartment syndrome. A high index of suspicion for vascular compromise, which is often misdiagnosed as compartment syndrome, must be maintained in the setting of tibiofemoral dislocation. Immediate manual reduction (D) is the intervention of choice in almost all tibiofemoral dislocations. The presence of a dimple sign, however, is a contraindication to manual reduction. One Step Further Question: What nerve is most commonly damaged in tibiofemoral dislocations? Answer: The common peroneal (fibular) nerve.

A 45-year-old woman presents to your office with right knee pain. She was playing volleyball yesterday when she collided with another player and was unable to continue playing. The knee was swollen this morning. She is able to walk but it is painful, and she also has pain when she attempts to bend her knee. On examination you have the patient stand flat footed on her right leg while you provide your shoulder for balance. You have her then flex her right knee to 20 degrees and rotate the femur on the tibia medially and laterally. This motion causes significant pain for the patient. You also note medial joint line tenderness. Which one of the following is the most likely cause of her knee pain? A Anterior cruciate ligament tear B Collateral ligament tear C Medial meniscus tear r D Tibial plateau fracture

Correct Answer ( C ) Explanation: A medial meniscus tear is the most likely diagnosis in a patient older than 40 years of age who was bearing weight when the injury occurred, was unable to continue the activity, and has a positive Thessaly test. This test is performed by having the patient stand on one leg and flex the knee to 20°, then internally and externally rotate the knee. Meniscus tears typically occur as a result of twisting or change of position of the weight-bearing knee in varying degrees of flexion or extension. The presence of swelling due to an effusion is commonly seen immediately after the injury. Pain from meniscus injuries is commonly intermittent and usually the result of synovitis or abnormal motion of the unstable meniscus fragment. The pain is usually localized to the joint line. Magnetic resonance imaging is the criterion standard imaging study for imaging meniscus pathology and all intra-articular disorders. Most meniscal tears do not heal without intervention. If conservative treatment does not allow the patient to resume desired activities, his or her occupation, or a sport, surgical treatment is considered. Surgical treatment of symptomatic meniscal tears is recommended because untreated tears may increase in size and may abrade articular cartilage. Tibial plateau fractures (D) are caused by a varus or valgus force combined with a weight bearing force on the knee. The classically described situation in which this occurs is from a car striking a pedestrian's fixed knee known as a "bumper fracture". This patient is able to bear weight, so a fracture is not likely. Anterior cruciate ligament (A) injuries are most often a result of low-velocity, noncontact, deceleration injuries and contact injuries with a rotational component. Quick stops and sharp cuts or turns create significant deceleration forces that can sprain or rupture the anterior cruciate ligament. The Lachman test is the most sensitive test for acute anterior cruciate ligament rupture. It is performed with the knee in 30° of flexion, with the patient lying supine. The amount of displacement and the quality of endpoint are assessed (firm, marginal, soft). Asymmetry in side-to-side laxity or a soft endpoint is indicative of an anterior cruciate ligament tear. The medial collateral ligament is most commonly injured as a result of direct lateral force to the knee such as clipping in football; this force creates a valgus load on the knee joint and can result in rupture of the medial collateral ligament. Conversely, a medial blow that creates a varus load can injure the lateral collateral ligament. The varus and valgus tests are sensitive tests to assess for medial and lateral collateral ligament tears (B).

An 18-year-old hurdler gets tripped-up and falls, contacting the running-track through his hyperextended left wrist. He presents to the ED with dorsolateral wrist pain, erythema, edema and anatomic snuffbox tenderness. Orthopedic consultation is obtained. Initial AP, lateral and oblique radiographs are negative for fracture. He is splinted and discharged. His discharge paperwork should contain which of the following directions? A Obtain MRI in four weeks if pain persists B Obtain primary-care medical clearance examination for upcoming orthopedic surgery C Obtain repeat radiographs in two weeks D Obtain rheumatological consultation for wrist inflammation

Correct Answer ( C ) Explanation: Falling on an out-stretched hand (FOOSH injury; "outstretched" refers to wrist hyperextension) is a common cause of scaphoid fracture, which represents the most common fractured carpal bone. The most common fracture site on the scaphoid is the wrist. Since the blood supply to the scaphoid enters distally, any >1 mm fracture-displacement is worrisome for being complicated by osteonecrosis and nonunion. A scaphoid fracture should be suspected any time there is snuff box tenderness. Wrist and scaphoid X-rays may be negative in the setting of acute fracture. Therefore, patients with snuff box tenderness or suspicion for scaphoid fracture should be placed in a thumb spica splint with repeat radiographs or MRI in 10-14 days. Waiting four weeks for repeat MRI (A) or plain radiographs is too long. Repeat imaging should occur within 10-14 days. It is unknown if a true fracture is present at this time, as such, surgery (B) would not be arranged at discharge. This wrist erythema and edema is related to trauma, and not to an underlying rheumatologic condition (D). One Step Further Question: Osteonecrosis is more likely with fractures of the scaphoid in the proximal or distal pole? Reveal Answer: The more proximal the fracture, the greater likelihood of osteonecrosis Rapid Review Scaphoid Fracture FOOSH Snuff box tenderness Possibly negative radiographs Complication: avascular necrosis Thumb spica splint

Rupture of the flexor digitorum superficialis tendon will result in the loss of what function? AFlexion at the distal interphalangeal joint BFlexion at the metacarpophalangeal joint CFlexion at the proximal interphalangeal joint DPincer function

Correct Answer ( C ) Explanation: Injuries to the flexor tendons in the hand are typically the result of lacerations, but closed traumatic disruption (e.g. avulsion fractures) can also occur. Careful examination of the hand with specific knowledge about the function of various tendons is essential as presentations can be subtle. The flexor digitorum superficialis (FDS) originates from the medial epicondyle of the humerus and travels distally through the anterior compartment of the forearm. At the wrist, four tendons come off the muscle and travel through the carpal tunnel. The tendons insert at the proximal aspect of the middle phalanx of each of the four fingers. The flexor digitorum profundus tendon passes deep to the split of the distal flexor digitorum superficialis tendon and inserts at the base of the distal phalanx. The primary function of the FDS is flexion at the proximal interphalangeal joint. On examination, the patient should be asked to flex at the PIP joint against resistance while the other three fingers are held in extension. Injuries to flexor tendons should be referred to a hand specialist for definitive management. If one is not immediately available, open wounds should be irrigated and approximated, the hand should be splinted in partial flexion at the MCP, PIP, and DIP joints and the patient should be referred for urgent outpatient management. The flexor digitorum profundus is responsible for flexion at the distal interphalangeal joint (A) and flexion at the metacarpophalangeal joint (B). The pincer function (D) also requires an intact flexor digitorum profundus in the finger. To test the function of the flexor digitorum profundus, the patient should attempt to flex the distal interphalangeal joint against resistance while holding the MCP and PIP joints in extension. One Step Further Question: What nerve innervates the flexor digitorum superficialis? Answer: The median nerve

Which of the following maneuvers tests the meniscus of the knee? A Anterior drawer test B Lachman test C McMurray test D Posterior drawer test

Correct Answer ( C ) Explanation: Meniscal injuries occur frequently in patients with sudden rotary or extension-flexion motions. The menisci have no sensory nerve fibers, and the pain that results after these injuries is from irritation of the ligaments near the joint line. Several symptoms suggest the presence of a meniscal tear including: joint line pain, joint effusion, locking, and giving way of the knee. The McMurray test is performed with the patient supine and the hip and knee flexed. To check the medial meniscus, the examiner palpates the posteromedial joint line with one hand while the other hand grasps the foot. The leg is externally rotated to trap the medial meniscus, and the knee is slowly extended. Conversely, the lateral meniscus is examined with the clinician palpating the posterolateral joint line while internally rotating the leg. A painful click, popping, or thud felt in early extension is considered abnormal. It is 53% sensitive for a meniscal injury. The Anterior Drawer (A) and Lachman tests (B) are used to assess anterior cruciate ligament (ACL) instability. The anterior drawer test is 62% sensitive for ACL disruption. With the affected knee flexed at 45 degrees, the clinician pulls the tibia forward away from the femur. The test is positive if the tibia is displaced more than 6 millimeters compared to the opposite leg. The Lachman test is more sensitive for ACL disruption (84%). The patient's affected knee is flexed 30 degrees. The clinician stabilizes the femur with one hand while the other hand grasps the tibia. The tibia is displaced in a forward direction. The test is positive if the tibia can be displaced greater than 5 millimeters anteriorly compared to the opposite leg. The Posterior Drawer test (D) assesses the posterior cruciate ligament. It is similar to the anterior drawer test, but instead of a forward displacement, the tibia is displaced posteriorly. It is 55% sensitive for PCL disruption. One Step Further Question: What is the most common injury associated with traumatic hemarthrosis of the knee joint? Answer: Anterior cruciate ligament (ACL) rupture. Rapid Review Meniscus Injuries Knee rotary stress Rapidly changing directions, squatting Medial > lateral Clicking, locking McMurray test Medial joint line pain: medial meniscus injury Lateral joint line pain: lateral meniscus injury Ege test: pain/click on knee rotation with patient in squatting position External rotation pain: medial meniscus injury Internal rotation pain: lateral meniscus injury Dx: MRI

A 70-year-old woman presents to clinic complaining of a dull, aching pain in her right knee that has been gradually worsening over the past two years. This pain was previously controlled with ibuprofen, but now her pain has started to limit her daily activities. What is the next best step in her treatment? A Acetaminophen B Glucosamine and chondroitin C Intra-articular injection of cortisone with lidocaine D Total knee arthroplasty

Correct Answer ( C ) Explanation: Osteoarthritis is a common malady that affects over 26.9 million Americans, and is one of the most common causes of long-term disability in the United States. There are varied risk factors for developing osteoarthritis including age, mechanical stress, and metabolic disorders such as ochronosis. Initial pharmacologic therapy consists of oral analgesics, such as acetaminophen or NSAIDs, but neither are recommended for long-term use. Steroid injections have been found to provide more long-term relief than oral therapies, and may be repeated up to every three months. More frequent injections of steroids can further exacerbate joint degeneration. The correct technique involves inserting the needle between the patella and femoral condyles with either a medial or lateral approach. The use of a local anesthetic (e.g. lidocaine) along with cortisone aids patient tolerance of the pain associated with intra-articular injection, and provides temporary pain relief while the steroid takes effect. For this patient who has failed oral therapy, an intra-articular injection of cortisone and lidocaine into the knee joint is recommended. While acetaminophen (A) is often recommended for initial management of osteoarthritis, it is unlikely to help in this patient's case as she has already failed treatment with ibuprofen. The use of glucosamine and chondroitin (B) for osteoarthritis has been controversial, and these widely used remedies are of uncertain benefit. A total knee arthroplasty (D) may be eventually necessary for this patient, but it is not necessary if there are other, less invasive, treatments available. One Step Further Question: Heberden nodes refer to what physical examination finding? Answer: Nodules on the distal interphalangeal joints of the hands associated with osteoarthritis.

A runner presents with recurrent anterior knee pain. She reports a "popping" sensation followed by severe pain. These episodes are brief because the pain resolves once she moves her "kneecap back into position". In the initial evaluation of this patient, a radiologic order would most likely include which of the following views? A Frogleg view B Grashey view C Sunrise view D Swimmer's view

Correct Answer ( C ) Explanation: Patellofemoral instability is the transient displacement (usually laterally) of the patella, either partially (subluxation) or completely (dislocation), causing acute or chronic patellar pain. It is associated with a positive apprehension sign when displacing the patella laterally. Diagnosis necessitates AP, lateral, tunnel and axial views. The axial view, also commonly called the sunrise view, allows the clinician a direct visualization of how the patella sits in the femoral trochlear groove. Normally, the "train is on the tracks", in other words, the patella sits equidistantly between the femoral condyles. In patients with malalignment or instability, the "train is off the tracks", showing the patella riding, most commonly, over the lateral femoral condyle, outside of the trochlear groove. Acute cases are treated with rest, icing, splinting and compression, but may also require manual reduction. Chronic recurrent cases require aggressive physical therapy and intermittent bracing. The frogleg lateral view (A) is used in evaluating the hip, not the knee. The Grashey view (B) is used in evaluating the glenohumeral joint, not the knee. A swimmer's view (D) is used to evaluate the cervicothoracic junction, not the knee. One Step Further Question: What are two common complications of untreated recurrent patellar instability? Answer: Quadricep weakness and patellar arthrosis

An 82-year-old woman with osteoporosis falls against her kitchen table. She presents with acute right shoulder pain, proximal right upper extremity edema and near-absent active shoulder joint range of motion. Which of the following is the most appropriate next step in management? AObtain a scapular-Y radiograph BPassive range of motion measurement CPerform a distal neurovascular examination DPerform a Hawkins testing

Correct Answer ( C ) Explanation: Proximal humerus fractures are most commonly seen in elderly patients. The most common type of fracture occurs at the surgical neck (the epiphyseal-diaphyseal junction, just inferior to both tuberosities). As with any fracture, it is imperative to assess distal neurovascular supply with motor, sensory, and pulse examination. Complications from shoulder injuries include injury to the brachial plexus and vascular such as the axillary artery. The Hawkins test (D), which usually follows Neer's testing of the shoulder, is used to evaluate subacromial impingement in the setting of rotator cuff syndrome or tear, the results of which are most likely noncontributory given this presentation. Passive range of motion measurements (B) will most likely be noncontributory and difficult to obtain. Radiographs (A) occur after the clinician is assured that no acute neurovascular compromise is present. One Step Further Question: The greater tuberosity is the insertion point of which 3 rotator cuff muscles? Reveal Answer: Supraspinatus, infraspinatus and teres minor. Rapid Review Proximal Humerus Fracture Neer classification: based on number of displaced (> 1 cm) or angulated (> 45°) parts Most common fracture: one part fracture Orthopedic consultation for displaced fractures

Which of the following fractures is most commonly non-operative in adults? A Galeazzi B Monteggia C Radial head fracture D Tibial plateau fracture

Correct Answer ( C ) Explanation: The majority of patients with a radial head fracture will not require operative management. Radial head fractures are typically caused by indirect trauma (e.g. fall on an outstretched hand). There are four types of radial head fractures ranging from the more common type I (undisplaced fracture) and type II (marginal fracture with minimal displacement) to the rarer type III (comminuted fracture) and type IV (fracture with dislocation). Type I and II injuries are usually treated with a sling and range of motion exercises. It is important to fully test the patient's range of motion. If the range of motion is limited by pain, the joint should be injected with an anesthetic agent (lidocaine, bupivicaine) and tested again. Continued limited range of motion in spite of pain control suggests the presence of entrapped fragments and orthopedic surgery may be required. Galeazzi (A) and Monteggia (B) fractures of the forearm have poor outcomes with nonsurgical management. The majority of tibial plateau fractures (D) also require surgery for acceptable functional outcomes. One Step Further Question: In what age group are supracondylar fractures common? Reveal Answer: Rapid Review Radial Head Fractures Type I/II: nonoperative Surgery indications (rule of 3s): >3 mm displacement, >30% of articular surface involvement, >30% angulation exists

A 49-year-old woman presents with a lump on the dorsum of her wrist which has been present for several weeks. She denies trauma. On examination, there is a soft, nontender, mobile mass on the dorsum of the wrist over the scapholunate joint. Which of the following is the most likely diagnosis? A Enchondroma B Felon C Ganglion cyst D Glomus tumor

Correct Answer ( C ) Explanation: The patient has a ganglion cyst, the most common soft tissue tumor of the hand. Ganglion cysts are filled with synovial fluid and develop either from a joint or the synovial lining of a tendon. They occur most commonly in the wrist and the flexor tendon sheaths of the fingers. They develop gradually and are often asymptomatic, though patients may complain of mild pain or paresthesias if the ganglion compresses a nerve. The lesions are benign, but treatment may be necessary for large or painful cysts. Ganglion cysts can be aspirated, but often recur. Definitive treatment is elective operative excision. Enchondromas (A) are the most common primary bone tumor in the hand, but would present as a firm deformity over a bone, not a soft mobile lump. Felon (B) is infection of the pulp of the distal fingertip. Glomus tumors (D) are benign vascular lesions. The most common presentation is a painful, bluish discoloration in the subungual area. One Step Further Question: What is the name for herpes simplex infection of the distal finger? Answer: Herpetic whitlow. Rapid Review Ganglion Cyst Patient will be complaining of a mass in wrist, sometimes painful Most commonly caused by repetitive activity causing tear or degeneration in joint capsule or tendon synovial sheath Treatment is observation or needle aspiration Comments: most common soft tissue tumors of the hand

A 58-year-old woman who works on an assembly line complains of bilateral wrist pain for the last several months. She describes pain, numbness, and paresthesias in her thumb, index, and long fingers. Which of the following tests is most likely to be positive? A Adson's test B Finkelstein's test C Phalen's test D Tinel's sign

Correct Answer ( C ) Explanation: The patient has carpal tunnel syndrome. Symptoms of carpal tunnel syndrome include gradual onset of numbness, paresthesias, and pain in the thumb, index, and long fingers. Symptoms are often worse at night and after strenuous activity. Carpal tunnel syndrome is commonly caused by repetitive strain. It can also be seen after distal radius and carpal bone fractures and dislocations, or as a result of systemic conditions (e.g rheumatoid arthritis, hypothyroidism, pregnancy, and diabetes). Phalen's test has a sensitivity of 76% and specificity of 80% for carpal tunnel syndrome. The test is performed by having the patient fully flex the wrists and push them together with the hands facing downward. A positive test is elicited if the patient develops paresthesias or numbness in the median nerve distribution within 60 seconds. Tinel's sign is less sensitive that Phalen's sign for carpal tunnel syndrome. A positive test is demonstration of pain or paresthesias in the median nerve distribution when the median nerve is tapped on at the wrist. Cervical radiculopathy and thoracic outlet syndrome should also be considered as symptoms can mimic carpal tunnel syndrome. Nerve conduction studies are used to confirm the diagnosis of carpal tunnel syndrome, with reported sensitivity of 85 to 90%. Nonoperative management for carpal tunnel syndrome includes splinting the wrist in a neutral position, antiinflammatory medications, and cortisone injections into the carpal tunnel. For persistent or severe symptoms, surgical release of the flexor retinaculum is performed. Adson's test (A) is used to assess for thoracic outlet pathology. Finkelstein's test (B) is pathognomonic for de Quervain's tenosynovitis. Tinel's sign (D) is less sensitive for carpal tunnel syndrome than Phalen's test. One Step Further Question: How is Finkelstein's test performed? Answer: The examining physician grasps the patient's thumb and ulnar deviates the hand sharply.

Which of the following is the most commonly fractured carpal bone seen in hand fractures? ACapitate BHamate CScaphoid DTriquetrum

Correct Answer ( C ) Explanation: The scaphoid is located in the proximal carpal row along with the lunate, triquetrum and pisiform bones. It is the most commonly fractured carpal bone. Mechanism of injury is generally a fall on an outstretched hand with the wrist in extension and radial deviation. Patients present with pain and swelling on the radial side of the wrist, limited range of motion and tenderness to palpation of the anatomic snuffbox. Initial management includes accurate diagnosis of the fracture, pain control, reduction and splinting (thumb spica splint) of the injury and referral to a hand surgeon for follow up. The capitate (A) and hamate (B) bones are both located in the distal carpal row. Fractures to bones in this row are less common than to those in the proximal row. The triquetrum (D) is located in the proximal carpal row and is the second most common carpal bone fracture. One Step Further Question: True or false: Metacarpal fractures are more common in adults than children? Answer: True

A 26-year-old man is training for a marathon and presents to the Emergency Department with right knee pain. The pain began during a run yesterday and has become progressively worse. He is unable to ascend stairs without significant pain. Palpation below the medial joint line of the knee elicits pain. You suspect anserine bursitis. The tendon of which of the following muscles is affected with this diagnosis? A Biceps femoris B Gastrocnemius C Gracilis D Vastus medialis

Correct Answer ( C ) Explanation: The tendons of the gracilis, sartorius and semitendinosus muscles insert onto the medial tibia below the knee joint and overlie the pes anserinus (Latin for "goose's foot") bursa. Pes anserine bursitis occurs commonly in runners, obese women with osteoarthritis and in other overuse syndromes. It is thought to be due to tight hamstrings, leading to increased friction and irritation of the bursa. Patients complain of anterior medial knee pain below the joint line and tenderness to palpation over the bursa. Symptoms may worsen with overuse or with certain activities such as squatting, running, or ascending or descending stairs. It usually is not symptomatic when walking on flat surfaces. Diagnosis is clinical. Management consists of rest, ice, proper stretching and nonsteroidal anti-inflammatory medications. The condition is self-limiting, and most athletes are able to return to play after a period of conservative therapy. The biceps femoris (A) muscle is part of the hamstring muscle group that arises from the ischium and femur and attaches to the lateral aspect of the tibia and fibula. The gastrocnemius (B) muscle has medial and lateral heads that originate on the medial and lateral femoral condyles, respectively, and attach to the calcaneus via the calcaneal (Achilles) tendon. The vastus medialis (D) muscle is a quadriceps muscle of the anterior thigh that extends the knee. Its tendon joins those of the other quadriceps muscles to form the quadriceps tendon that attaches to the anterior proximal tibia. One Step Further Question: What concomitant knee injury should be evaluated for in patients with pes anserine bursitis? Answer: Medial collateral ligament injury (strain or tear), due to its insertion site near the bursa

A 59-year-old woman presents to clinic with complaints of right lateral hip pain which has been ongoing for four months. She is unable to sleep on her right side secondary to the pain. The patient has pain and tenderness with palpation of her greater trochanter. There is no erythema and no soft tissue swelling. The patient has tried nonsteroidal anti-inflammatory drugs and physical therapy for the last six weeks but her pain persists. What is the most appropriate next step in management? A Amitriptyline B Clindamycin C Corticosteroid injection D Trochanteric bursectomy

Correct Answer ( C ) Explanation: This patient has greater trochanteric pain syndrome (previously known as trochanteric bursitis) which typically presents as lateral hip pain which is worse with lying on the affected side, and tenderness with palpation of the greater trochanter. Greater trochanteric pain syndrome is the most common cause of lateral hip pain in adults. First-line treatment is conservative therapy, including nonsteroidal antiinflammatory drugs (NSAIDs) and physical therapy. If pain does not adequately respond to these interventions, a corticosteroid injection to the trochanteric bursa is recommended. This can be done with or without ultrasound guidance. The injection can provide both diagnostic and therapeutic benefit. Amitriptyline (A) is useful for neurogenic pain and is appropriate management for fibromyalgia. However, in this patient with an isolated finding of unilateral right hip pain, fibromyalgia is a less likely diagnosis. Clindamycin (B) is appropriate for treatment of septic bursitis. This patient's history and clinical exam, chronicity of pain and lack of erythema and soft tissue swelling, are more consistent with aseptic bursitis. Trochanteric bursectomy (D) is only considered after conservative management with NSAIDs, physical therapy, and corticosteroid injections fails. One Step Further Question: What is the prognosis for greater trochanteric pain syndrome? Answer: The syndrome is generally self-limiting and resolves spontaneously

A 64-year-old man complains of pain and paresthesias in his right hand intermittently for several weeks. He works in a factory putting together electronics. On exam, he has decreased sensation of his right 1st through 4th digits and an atrophied thenar eminence. What test in the ED will help diagnose his condition? A CT scan of the head B Electromyelography (EMG) C Percuss the right volar wrist D Urine drug screen

Correct Answer ( C ) Explanation: This patient has median mononeuropathy, also known as carpal tunnel syndrome, a compression neuropathy of the median nerve as it traverses under the flexor retinaculum at the wrist. The median nerve provides sensation primarily to the palmar aspect of the 1st, 2nd, 3rd, and radial side of the 4th. When it is compressed, the patient experiences pain, paresthesias, and numbness in that distribution. The Tinel's test is performed by lightly tapping the volar surface of the wrist over the median nerve. This should elicit a sensation of tingling or pins and needles in the distribution of the median nerve. Carpal tunnel syndrome is first treated with wrist splinting and initiation of a more ergonomic work environment. NSAIDs may also be helpful. If symptoms do not improve, the patient should be referred to a hand specialist who may elect to perform a carpal tunnel release procedure. CT scan of the head (A) would be useful if there is suspicion that this patient's symptoms are from a central process such as an acute stroke. However, his numbness is in a peripheral nerve distribution and not dermatomal. An EMG (B) is used most commonly by neurologists to confirm damage to peripheral nerves. It may be used for carpal tunnel syndrome if the symptoms do not resolve with conservative management. A variety of heavy metals are associated with a peripheral neuropathy (lead, mercury), but these require special serologic testing, not a urine drug screen (D). The patient is at risk for occupational exposures because he works in a factory; however, most heavy metal poisonings are associated with other symptoms. One Step Further Question: What is the Phalen maneuver? Answer: The opposing dorsal surfaces of the hands are pressed together with the wrists flexed for 60 seconds. It is positive for median nerve problems if this reproduces or worsens symptoms. Rapid Review Carpal Tunnel Syndrome Durkan's test: reproduction of symptoms with carpal tunnel compression Phalen's sign: reproduction of symptoms with wrist hyperflexion Tinel's sign: reproduction of symptoms with percussion over carpal tunnel Flick sign: patient reports shaking of the hand provides relief NSAIDs, volar splint in neutral position

A 23-year-old man presents with swelling and pain of his elbow. On examination, he has swelling over the olecranon. He has full range of motion at the joint. Vital signs are normal. What management is indicated? A Arthrocentesis B Aspiration C Compression dressing and nonsteroidal anti-inflammatory drugs Correct Answer D Incision and drainage

Correct Answer ( C ) Explanation: This patient presents with a noninfectious olecranon bursitis that should initially be managed with a compression dressing, ice and NSAIDs. The olecranon bursa frequently becomes inflamed as a result of repetitive minor trauma. Patients will typically present with pain, swelling and tenderness over the olecranon. The inflammation is often obvious on physical examination. Although active range of motion may be limited by pain, patients should have full passive range of motion as the joint is not involved. Noninfectious bursitis should have minimal or no warmth and erythema. Noninfectious bursitis should be treated conservatively with compression dressings, ice, NSAIDs and orthopedic follow up as well as avoidance of the inciting trauma. Arthrocentesis (A) should only be performed if it is believed that the joint is involved. Aspiration (B) of the bursa may be therapeutic and assists in the diagnosis of septic bursitis; but it is not a requirement in noninfectious bursitis. Incision and drainage (D) is reserved for recurrent or refractory bursitis. One Step Further Question: What organisms are typically implicated in septic bursitis? Answer: Usual skin flora

A 63-year-old man with hypertension and dyslipidemia presents with pain and swelling of his left great toe. He denies trauma or fever. Examination reveals an exquisitely tender and swollen first metatarsophalangeal (MTP) joint on his left foot. Arthrocentesis yields fluid as shown above. What is the first line therapy for this patient? A Acetaminophen B Allopurinol C Ibuprofen D Probenecid

Correct Answer ( C ) Explanation: This patient presents with monoarticular arthritis and a synovial fluid sample consistent with gout. Gout is a systemic disorder that manifests with joint inflammation. It is caused by precipitation of uric acid crystals from extracellular fluid. Hyperuricemia results either from underexcretion of end products of purine metabolism from the kidney or overproduction (more rare). During an episode of gout, polymorphonuclear cells ingest the crystals and release cytokines leading to an inflammatory reaction within the synovium. The most commonly affected joint is the first metatarsophalangeal (MTP) joint followed by the knee. It is important to consider that patients with gout are at a higher risk of developing septic arthritis because the joints are chronically damaged. In patients with a history of gout that present with symptoms concerning for septic arthritis (pain, fever, decreased range of motion) arthrocentesis should be performed to rule out the presence of infection. Management of gout should be split into acute gouty attack treatment and long-term prophylaxis. The mainstay of acute therapy is NSAIDs and colchicine. Indomethacin, naproxen and ibuprofen all may be used in acute treatment. Colchicine inhibits microtubule formation reducing the inflammatory response to uric acid crystals. Unfortunately, the drug has a number of side effects that are almost universally experienced at therapeutic doses (nausea, vomiting, diarrhea). Acetaminophen (A) does not have anti-inflammatory properties and offers little benefit in acute gouty attacks. Allopurinol (B) is a xanthine oxidase inhibitor that prevents production of uric acid. It is useful in patients with increased synthesis and decreased clearance of uric acid. Allopurinol should not be started at the time of an acute attack of gout because it can lead to a transient increase in uric acid levels. However, patients already prescribed allopurinol should continue to take it at the same dose during acute episodes. Probenecid (D) is useful in prophylaxis but not for acute episodes. One Step Further Question: What finding is seen on synovial fluid analysis in pseudogout? Answer: Positively birefringent, rhomboid-shaped crystals. Rapid Review Gout Patient will be a middle-aged man Complaining of acute onset of pain in the first MTP (Podagra) Labs will show needle-shaped crystal with negative birefringence Most commonly caused by uric acid crystals Treatment is: Acute: NSAID's Chronic: allopurinol or colchicine Comments: can be triggered by loop and thiazide diuretics

A 16-year-old girl presents with pain to the right index finger after slamming it in a car door. Physical examination reveals a 50% subungal hematoma and intact nail folds. An X-ray shows a non-displaced distal tuft fracture. What management is indicated? A Oral antibiotics, splinting and follow up B Splinting and follow up C Trephination, splinting and follow up D Trephination, splinting, oral antibiotics and follow up

Correct Answer ( C ) Explanation: This patient should have trephination performed to relieve pain from the subungal hematoma, be splinted in extension and follow up with a hand surgeon. Subungal hematomas are common after crush injuries to the digits. They are often associated with distal phalanx fractures. Pain associated with these hematomas can be severe as pressure increases under the nail. Management focuses on diagnosing associated injuries and providing pain relief. The easiest way to relieve pain in the digit is by relieving the pressure building under the nail. This can be accomplished with trephination of the nail. Typically, an 18-gauge needle or electrocautery device is placed over the center of the hematoma. Gentle pressure is applied until the nail is penetrated. This typically results in a drop or two of blood exiting through the hole that has been created. The patient typically experiences immediate pain relief. Nail removal is typically unnecessary. Administration of antibiotics (A) has not been shown to be helpful to prevent infection. Splinting and follow up (B) are appropriate after trephination. Even though trephination in the presence of a distal phalanx fracture technically converts a closed fracture into an open one, it is not contraindicated to perform drainage of the blood. Prophylactic antibiotics (D) are not necessary. As usual, tetanus prophylaxis should be administered as needed. One Step Further Question: When should the nail be removed in a patient with a subungal hematoma? Reveal Answer: Rapid Review Trephination, Nail Indication: acute, not spontaneously draining, intact nail folds, painful Methods: handheld electrocautery (preferred), 18-gauge needle/syringe Post procedure care: keep clean and dry for 2 days

A 54-year-old woman presents with a swollen knee. On examination, a large joint effusion is present. With which of the following spaces does the knee joint communicate? A Anserine bursa B Infrapatellar bursa C Prepatellar bursa D Suprapatellar bursa

Correct Answer ( D ) Explanation: A knee effusion is defined as fluid within the knee joint. Symmetric peripatellar or suprapatellar swelling will be present. In large effusions, excess fluid elevates the patella above the femur and the patella is ballottable against the femur. Effusions can be due to excess synovial fluid from traumatic or atraumatic inflammation, or hemarthrosis following trauma to the knee. Traumatic effusions may result from ligamentous, bony, or meniscal injuries or overuse syndromes. Atraumatic etiologies of knee effusions include osteoarthritis, crystal arthropathies (e.g. gout and pseudogout), and septic arthritis. The knee has several bursa, which are small fluid-filled sacs which serve to decrease friction between moving structures. Some of the bursa communicate with the joint cavity directly and some do not. The suprapatellar bursa, the largest of the bursae, is not a true bursa but rather an extension of the knee joint capsule. When a knee effusion or hemarthrosis is present, fluid can freely flow into and distend the suprapatellar bursa. One technique to increase detection of small effusions is to "milk" the suprapatellar bursa, forcing fluid back into the knee joint. The anserine bursa (A) is located beneath the anserine tendon where the gracilis, sartorius, and semitendinosus muscles insert. It does not communicate with the knee joint. There are two infrapatellar bursae (B), a superficial infrapatellar bursa and a deep infrapatellar bursa, which, in most individuals, do not communicate with the joint space. The prepatellar bursa (C) is located between the patella and the skin, and does not communicate with the joint space. One Step Further Question: What condition is caused by prolonged or frequent kneeling? Answer: Prepatellar bursitis

A 55-year-old man presents to the ED with left knee pain after a fall from standing 2 days prior. He has a history of diabetes mellitus. He states he has been progressively less able to walk because of the pain. On physical examination, the left knee is warm, erythematous, diffusely tender, and swollen with a large effusion. There are no rashes or deformities. He has significant pain on passive range of motion. His vital signs are BP 150/85 mm Hg, HR 105 beats per minute, RR 16 breaths per minute, and T 102.7°F. Which of the following synovial fluid analysis results is most indicative of a diagnosis of septic arthritis? A 40,000 WBC per high power field with 50% neutrophils and glucose 50 mg/dL B 400 WBC per high power field with 20% neutrophils and glucose 95 mg/dL C 5,000 WBC per high power field with 80% neutrophils and glucose 80 mg/dL D 80,000 WBC per high power field with 90% neutrophils and glucose 40 mg/dL

Correct Answer ( D ) Explanation: This patient is exhibiting the common signs and symptoms of septic arthritis, an infection of a joint space. This is most commonly seen in the knee, followed by the hip, shoulder, and wrist. Risk factors for the development of septic arthritis include, but are not limited to, pre-existing arthritis, immunocompromised states, and high-risk sexual practices. Patients typically present with a warm, tender, erythematous, swollen joint and pain with passive range of motion. Fever and chills are also common. Arthrocentesis is the necessary diagnostic procedure to obtain synovial fluid, which is the only way to definitively diagnose septic arthritis. Staphylococcus aureus remains the predominant pathogen for all age groups; however, in sexually active young adults, Neisseria gonorrhoeae is common and should also be included in the selection of antibiotic coverage until culture results from the synovial fluid return. In septic arthritis, the synovial fluid is typically cloudy, has a high WBC (> 50,000 WBC per high power field), a predominance of neutrophils, and a low glucose when compared to serum. The synovial fluid with 80,000 WBC per high power field with 90% neutrophils and glucose 40 mg/dL is most consistent with the diagnosis of septic arthritis. Definitive management of septic arthritis typically includes both IV antibiotics and operative washout of the affected joint by an orthopedic surgeon. 400 WBC per high power field with 20% neutrophils and glucose 95 mg/dL (B) is representative of what would be seen in the synovial fluid of a patient with degenerative joint disease or osteoarthritis. 5,000 WBC per high power field with 80% neutrophils and glucose 80 mg/dL (C) is representative of what would be seen in the synovial fluid with an episode of acute gout. The synovial fluid in acute gout characteristically has 2,000-5,000 WBC per high power field with >75% neutrophils. The glucose level in the synovial fluid is typically similar to that of the serum during an episode of acute gout. Gout is also typified by the presence of negatively bifringent needle-shaped crystals. 40,000 WBC per high power field with 50% neutrophils and glucose 50 mg/dL (A) is representative of what would be seen in the synovial fluid of a patient with inflammatory arthritis, such as rheumatoid arthritis. The synovial fluid of inflammatory arthritis is characterized by 5,000-50,000 WBC per high power field with a composition of both neutrophils and lymphocytes. The glucose level in the synovial fluid is often lower than that of the serum; however, not quite as low as that which is seen with septic arthritis. One Step Further Question: What do the crystals seen in the synovial fluid of a patient with a diagnosis of pseudogout consist of? Answer: Calcium pyrophosphate

Which of the following is a complication of an untreated mallet finger injury? A Boutonniere deformity B Dupuytren's contracture C Jersey finger D Swan-neck deformity

Correct Answer ( D ) Explanation: A mallet finger is caused by a forced flexion of the distal interphalangeal joint (DIP) leading to rupture of the extensor tendon at its insertion at the base of the distal phalanx or bony avulsion of the tendon insertion site. Appropriate treatment includes splinting the distal interphalangeal joint in full extension or slight hyperextension while allowing the proximal interphalangeal joint to have full range of motion. In untreated or under treated cases, a swan neck deformity occurs. Other causes include rheumatoid arthritis and connective tissue disorders (Ehlers-Danlos syndrome). A swan neck deformity is characterized by hyperextension of the proximal interphalangeal joint (PIP) and flexion of the distal interphalangeal joint (DIP). Dupuytren's contracture (B) refers to a thickened and fibrous palmar fascia which is due to fibrous proliferation. It occurs commonly in men over age 40 years, but it also is common in those of Northern European descent, diabetics, alcoholics and epileptics. Boutonniere deformity (A) occurs from forced flexion at the proximal interphalangeal joint (PIP), causing a tear of the central portion of the extensor tendon at the proximal interphalangeal joint (PIP). Patients are unable to fully extend at the proximal interphalangeal joint (PIP) with the wrist and metacarpalphalangeal joints fully extended. Jersey finger (C) occurs in cases such as a football player grabs another player's jersey, avulsing the profundus tendon from its bony insertion. One Step Further Question: As compared to Dupuytren's contracture, where is the pathology located in stenosing tenosynovitis (trigger finger)? Answer: Fibrotic nodules occurring in the digital flexor tendon

A 73-year-old man with a history of arthritis presents with complaints of a low-grade fever and severe right knee pain for the past three days, with an inability to bear weight since this morning. On exam, you note exquisite right knee tenderness and a large effusion. There is limited range of motion both actively and passively, and he refuses to ambulate. You perform an arthrocentesis and drain 20 mL of turbid fluid. Laboratory analysis of the joint fluid reveals the following: WBC of 55,000/µL with 95% neutrophils and a glucose level of 60 mg/dl (serum glucose is 140 mg/dl). Gram stain and crystal analysis are not immediately available. Which of the following is the most likely diagnosis? A Acute gout B Osteoarthritis C Rheumatoid arthritis D Septic arthritis

Correct Answer ( D ) Explanation: Any patient with an acute monoarticular arthritis should be considered to have septic arthritis until proven otherwise. Patients at increased risk for septic arthritis include the elderly, those with prosthetic joints, IV drug abusers, and the immunocompromised. Septic arthritis often occurs in patients with a history of chronic arthritis, complicating the diagnosis. In healthy adults, the knee is the most commonly affected joint, but in IV drug abusers, common sites include the sacroiliac, sternoclavicular, and intervertebral joints. In children, the knee and hip are most commonly affected. Synovial fluid results in this case are consistent with a bacterial rather than inflammatory etiology (see table below). Fluid culture will help confirm the diagnosis of septic arthritis, but such results take time to complete. Empiric treatment should be initiated. Gout (A) is a disease of middle-aged men and elderly adults and is typically monoarticular. It is caused by deposition of uric acid crystals and usually affects the lower extremities, particularly the great toe (podagra). Evaluation of synovial fluid reveals needle-shaped crystals within PMNs that are negative birefringence under polarized light. Osteoarthritis (B) is a degenerative joint disease that results in joint-space narrowing seen on radiograph. Rheumatoid arthritis (C) typically affects older women and most commonly occurs in the hand (metacarpal phalangeal and proximal interphalangeal joints), wrist, and elbow. One Step Further Question: What is the most common cause of septic arthritis? Answer: Staphylococcus aureus is the most common cause of septic arthritis in adults. Rapid Review Septic Arthritis Age < 35: N. gonorrhea S. aureus most common overall Hematogenous spread Fever, pain, ↓ ROM Knee (most common) Arthrocentesis (WBC > 50,000 with > 75% PMNs) IV ABX, surgical washout

A 12-year-old obese boy presents with groin and proximal anterior thigh pain. He has no pain below the knee. Physical examination shows a significant decrease in internal rotation of the affected hip. Radiographs show a widened growth plate. Which of the following is the most likely diagnosis? A Acute transient synovitis B Legg-Calve-Perthes disease C Osgood-Schlatter disease D Slipped capital femoral epiphysis

Correct Answer ( D ) Explanation: Atraumatic pediatric hip pain or limp is usually caused by acute transient synovitis, Legg-Calve-Perthes disease or slipped capital femoral epiphysis (SCFE). SCFE is a fracture in the physis (growth plate) of the femoral head which leads to slippage of the overlying epiphysis. It usually occurs during adolescent growth spurts. Predisposing factors include male sex, obesity and increased sports activities. The typical age range is 10-14 years for girls and 11-16 years for boys. Patients present with pain in the anterior proximal knee or thigh that is exacerbated by activity. On exam, there is loss of hip internal rotation, particularly with the hip flexed. Patients typically walk with the involved extremity externally rotated. AP and frog-leg radiographs show "ice cream falling off the cone." All cases warrant urgent orthopedic evaluation for stabilization surgery. Patients should be nonweight bearing and restricted to bed rest until then. Complications of untreated disease include chondrolysis and osteonecrosis. Acute transient synovitis (A) is considered the most common cause of childhood hip pain or limp. It is most common in boys aged 3-6 years. Radiographs are normal. Treatment includes NSAIDs and relative rest. Legg-Calve-Perthes (B) disease is avascular osteonecrosis of the femoral head. It occurs more commonly in boys aged 4-10 years. Radiographs show a small, sclerosed femoral head and widened joint space. Osgood-Schlatter (C) disease is a common adolescent condition in those active in sports. Repeated microtrauma occurs at the apophyseal cartilage between the anterior tibial tubercle and the secondary ossification center of the patellar tendon. These patients present with anteroinferior knee or superior shin pain and tender tibial tuberosities. One Step Further Question: Which endocrinopathies predispose adolescents to Slipped Capital Femoral Epiphysis? Answer: Hypothyroidism and growth hormone deficiency. Rapid Review Slipped Capital Femoral Epiphysis Obese African-American male adolescents (11-13) Left > right Limp, hip or groin pain Painful/↓ hip internal rotation AP/lateral X-ray of bilateral hips: abnormal Klein line, Bloomberg's sign Non-weight-bearing, orthopedic consultation

Which finding on synovial fluid analysis is most consistent with the diagnosis of pseudogout? A Elevated synovial leukocyte count B Multiple gram-negative cocci C Negatively birefringent urate crystals D Positively birefringent calcium pyrophosphate dihydrate crystals

Correct Answer ( D ) Explanation: Calcium pyrophosphate crystal deposition disease, also referred to as pseudogout, is a common crystal-induced arthropathy that generally affects the large joints. Pseudogout has a similar clinical presentation to gout, but the etiology is different. Pseudogout may be idiopathic, especially in the elderly. It may also be caused by trauma, hyperparathyroidism, hemochromatosis, and medications that cause hypomagnesemia such as loop diuretics or proton pump inhibitors used in peptic ulcer disease. Patients present with acute onset of severe pain, inflammation and edema in the knees, ankles, elbows or wrists. Pseudogout is generally monoarticular, but may present in multiple joints as well. Diagnosis is by synovial fluid analysis, so arthrocentesis is necessary for patients with monoarticular arthritis. Synovial fluid analysis will show positively birefringent calcium pyrophosphate dihydrate crystals, which confirms the diagnosis. Initial treatment for pseudogout is with nonsteroidal anti-inflammatory drugs (NSAIDs) or colchicine. Elevated synovial leukocyte count (A) is seen in septic arthritis. Multiple gram-negative cocci (B) aspirated from an inflamed joint points to the diagnosis of gonococcal arthritis, caused by infection with the sexually transmitted Neisseria gonorrhoeae bacteria. Negatively birefringent urate crystals (C) seen on synovial fluid analysis are consistent with the diagnosis of gout. One Step Further Question: What types of food or drink can bring on an attack of acute gout? Answer: Meat, seafood, alcohol and foods high in purine

30-year-old woman complains of anterior knee pain that gets worse with prolonged sitting, going up and down stairs and with deep squats. She has no known history of knee injury. She exercises three times a week on her exercise bicycle. She is average weight and played basketball as a high school student. Which of the following is the most likely diagnosis? A Chronic anterior cruciate ligament tear B Osgood-Schlatter disease C Osteochondritis dissecans D Patellofemoral syndrome

Correct Answer ( D ) Explanation: Patella-related pain is the single most common cause of knee pain. Patellofemoral pain syndrome is a multifactorial syndrome characterized by aching anterior knee pain that worsens with activities that stress the patellofemoral joint (climbing stairs, kneeling). Patients complain of diffuse, aching anterior knee pain that is exacerbated by loaded flexion activities such as stair climbing, jumping, or prolonged sitting (theater sign). On exam there may be patellar crepitation. The diagnosis is clinical and knee radiographs provide limited information (articular cartilage loss). Treatment includes activity modification and an exercise program consisting of quadriceps strengthening (medial quadriceps) and hamstring flexibility. NSAIDs are recommended for symptomatic care. Osteochondritis dissecans (C) is due to repetitive small stress to the subchondral bone that leads to osteonecrosis (most commonly the medial femoral condyle). Most cases begin in childhood although individuals may not become symptomatic until late adolescence or early adulthood. Early diagnosis is critical, as the injury has a better potential to heal while the bones are still growing. Osgood-Schlatter disease (B) is caused by rupture of the growth plate at the tibial tuberosity, which causes stress on the patellar tendon. It most commonly occurs in rapidly growing adolescents (10-15-years-old) and is five times more common among those active in sports and up to three times more common in boys. Chronic anterior cruciate ligament (ACL) tear (A) is a chronic injury seen in individuals who have a history of recurrent episodes of knee instability associated with swelling and limited motion. Patients may describe locking or a "giving way" phenomenon. They may also give a history of a remote injury to the knee that was not rehabilitated. One Step Further Question: What is the most common cause of an acute hemarthrosis after a sports-related knee injury? Answer: ACL tear. Rapid Review Patellofemoral Pain Syndrome Patient will be a woman Complaining of aching anterior knee pain, that is worse with loaded flexion (stair climbing, jumping, prolonged sitting) PE will show patellar crepitation Treatment is to strengthen medial quadriceps, NSAIDs

Which of the following is the most appropriate initial long term treatment for patellofemoral pain syndrome? A Cyclobenzaprine B Knee surgery C Non-steroidal anti-inflammatory drugs D Physical therapy

Correct Answer ( D ) Explanation: Patellofemoral pain syndrome (PFPS) describes pain around the anterior knee at the kneecap. It has many causes but is often related to poor biomechanics, imbalance in muscle strength, or drastic increases in physical activity level. Treatment consists of non-steroidal anti-inflammatory drugs and icing in the short term, and physical therapy in the long term. Goals of physical therapy include stretching properly and strengthening muscles specifically to improve alignment and tracking of the patella. Although non-steroidal anti-inflammatory drugs (C) are beneficial in the short term for pain control in PFPS they are not the most appropriate long term management option as this resolves symptoms, but does not cure the underlying imbalance in muscle strength and patellar alignment. Evidence does not support the use of muscle relaxants (A) in the treatment of PFPS. Surgery (B) would not be considered until 12 months of physical rehabilitation has failed. One Step Further Question: The patellar grind test is used to diagnose what condition? Answer: Patellofemoral pain syndrome

A 16-year-old girl presents to clinic with 3 weeks of worsening right knee pain. She attends dance class since age 5 and now dances 20 hours a week. She denies any inciting injury and continues to dance on the injured leg. On exam, she has pain at the inferior and medial pole of the right patella with no swelling or erythema of the knee. Which of the following is the most likely diagnosis? A Anterior cruciate ligament rupture B Medial meniscus tear C Patella dislocation D Patellofemoral pain syndrome

Correct Answer ( D ) Explanation: Patellofemoral pain syndrome occurs in about a quarter of those involved in athletics and more commonly in women and those between the ages of 10 and 35 years. Symptoms include pain with going up and down stairs and prolonged sitting or squatting. Individuals may have a sensation of the knee buckling or giving way. Swelling, popping or grinding sensations may be present. Treatments include non-steroidal anti-inflammatory medications, ice, quadriceps strengthening, stretching, patella bracing and orthotics. The anterior cruciate ligament (A) is usually injured by hyperextension, rapid deceleration or twisting movement and is characterized by early swelling and immediate severe pain. A meniscus tear (B) in young adults and adolescents usually occurs from a sudden flexing or twisting of the knee. A loose piece of cartilage can get stuck in the joint and cause temporary locking of the knee and prevention of full extension of the leg. Patella dislocation (C) is usually caused by sudden twisting of the leg or a direct blow that causes the patella to dislocate lateral of its normal placement in the patellofemoral groove. The patient experiences intense pain and typically has effusion on exam. One Step Further Question: What imaging modality is the gold standard for diagnosing an anterior cruciate ligament injury? Answer: MRI.

A 19-year-old softball player presents to clinic with complaints of bilateral knee pain which has been ongoing for approximately three months. She denies a history of trauma to either knee. The onset has been insidious and slowly increasing over time. She reports worsening of her pain when playing catcher and climbing stairs. She locates the pain over her anterior knees. X-rays are obtained that are normal. Which of the following is the most likely diagnosis? A Meniscus tear B Osgood-Schlatter disease C Osteochondritis dissecans D Patellofemoral syndrome

Correct Answer ( D ) Explanation: Patellofemoral syndrome or patellofemoral pain presents as anterior knee pain at or around the patella. The cause is not fully understood but may be associated with damage of the cartilage on the undersurface of the patella. The condition is more common in women than men and often presents initially in the adolescent or young adult. Symptoms are exacerbated by forces that increased the patellofemoral contact pressure such as deep knee bending, stair climbing, or prolonged sitting with the knee bent. Physical exam may reveal quadriceps atrophy, patellar maltracking, pain with compression of the patella, or palpable crepitus under the kneecap. A meniscus tear (A) is unlikely in a young patient who has bilateral involvement and no history of trauma. Osgood-Schlatter disease (B) is a traction apophysitis of the tibial tubercle at the insertion of the patellar tendon. It should be considered among the differential diagnosis in a younger patient presenting with insidious onset anterior knee pain; however, Osgood-Schlatter disease generally resolves as the apophysis fuses with the tibial tubercle and is unlikely in a patient of this age who has reached skeletal maturity. Osteochondritis dissecans (C) can present as vague and poorly localized activity-related knee pain. Osteochondritis dissecans is generally identified on X-rays as an area of subchondral lucency, best seen on the tunnel view (i.e. knee bent between 30-50 degrees). It should be considered as a differential diagnosis in this patient but is less likely given bilateral involvement, localized pain to the anterior knees, and normal X-rays. One Step Further Question: What is the treatment for patellofemoral syndrome? Answer: First-line treatment for patellofemoral syndrome is rest or activity modifications, NSAIDs, and physical therapy for quadriceps strengthening (specifically strengthening of the vastus medialis obliquus)

A 32-year-old man presents with acute onset of a hot, swollen left knee. The right knee and other peripheral joints are unaffected. Skin lesions are absent. Cardiopulmonary, genitourinary and neurologic examinations show no gross abnormalities. Laboratory examination reveals a leukocytosis. An X-ray shows normal joint space and absent osteophytes. Which of the following is the most likely diagnosis? A Lyme disease B Osteoarthritis C Rheumatoid arthritis D Septic arthritis

Correct Answer ( D ) Explanation: Septic, or pyogenic, arthritis usually occurs after direct inoculation or hematogenous spread into a joint. In adults, there may be an underlying arthritis, whether degenerative or rheumatologic, in the joint. Common causative organisms in adults are Staphylococcus aureus and Neiserria gonorrhoeae. Joint edema, warmth, decreased range and protected use are common symptoms. It is important during any examination of suspected septic arthritis to search for a primary source, such as a penetrating wound, local abscess or skin infection. Laboratory examination includes WBC, CRP and ESR. A joint aspirate (arthrocentesis) should be sent for culture, Gram stain and fluid analysis. Blood cultures are also routinely drawn. Broad-spectrum intravenous antibiotics are started immediately. Surgical decompression and drainage are considered if a patient doesn't respond to antibiotics in 24-48 hours, or if the affected joint is the hip. Common antibiotic choices include oxacillin, cefazolin, ceftriaxone and aminoglycosides. Lyme disease (A) presents with insidious onset, arthritis, rash and cardiac or neurologic manifestations. Osteoarthritis (B) has radiographic abnormalities of joint space narrowing and osteophytosis. Rheumatoid arthritis (C) is associated with an abnormal ESR, symmetric involvement, distal joint predilection and morning stiffness. One Step Further Question: Neonatal septic arthritis is commonly caused by which organisms? Answer: Staphylococcus aureus and group-B Streptococcus. Haemophilus influenzae should be considered in childhood septic arthritis

A 13-year-old obese boy presents with complaints of right anterior thigh pain and a limp for 2 weeks. There is no history of trauma. On exam, he is able to bear weight but walks with his right foot externally rotated. He has decreased motion of his right hip and when his hip is flexed, the extremity externally rotates. The patient is afebrile. White blood cell count is 8,000. Which of the following is the most likely diagnosis? A Developmental dysplasia of the hip B Legg-Calve-Perthes disease C Septic arthritis of the hip D Slipped capital femoral epiphysis

Correct Answer ( D ) Explanation: Slipped capital femoral epiphysis (SCFE) occurs when the head of the femur (epiphysis) slips relative to the femoral neck. This occurs at the point of weakness in the growth plate (physis). The single greatest risk factor is obesity and is more common in boys than girls. The average age is 12 for girls and 13 for boys. The patient often presents with a history of pain in the groin or anterior thigh although they may also present with referred pain in the knee. Patients usually present with a limp but may be unable to bear weight secondary to pain. On exam, the patients will have decreased range of motion of the hip and obligatory external rotation, where the leg externally rotates with hip flexion. X-rays demonstrate widening of the physis and slipped epiphysis on a lateral view of the hip, confirming the diagnosis. Developmental Dysplasia of the hip (A) is a congenital hip disorder in which there is abnormal development of the acetabulum and proximal femur. Hip dysplasia may present with a limp but this usually is apparent shortly after onset of walking and is generally painless at onset. Legg-Calve-Perthes Disease (B) is a form of avascular necrosis of the hip. Patients with Legg-Calve-Perthes present with a limp; hip or thigh pain may or may not be present. The peak incidence is in a younger patient, ages 4-8, and is more common in boys than girls. Septic arthritis of the hip (C) is a surgical emergency and should be considered but is unlikely in a patient that is afebrile, has a normal white blood cell count, and is able to bear weight. One Step Further Question: What is the treatment for Slipped Capital Femoral Epiphysis (SCFE)? Answer: The treatment for SCFE is prompt operative stabilization

A 63-year-old women presents to the emergency room with excruciating pain after falling in the shower. She had a hip replacement 3 months ago due to severe osteoporosis of her hip. On exam, her leg is internally rotated and shortened. An X-ray is obtained as seen in the image above. Which of the following is the most likely diagnosis? A Anterior dislocation of the hip B Femoral neck fracture C Hip fracture D Posterior dislocation of the hip

Correct Answer ( D ) Explanation: The X-ray shows a right-sided posterior hip dislocation. Dislocated hips are frequently seen in elderly patients and are considered medical emergencies. They are also more common in patients who have had a hip replacement. Dislocation occurs when the ball-shaped head of the femur comes out of the cup-shaped acetabulum set in the pelvis. Posterior (vs anterior) hip dislocations are the most common. On physical examination the limb is shortened, and the hip flexed, the foot is in internal rotation. Check for other fractures especially femur neck or shaft fractures and tibial fractures. Reduction should be done immediately. During a reduction, general anesthetic and muscle relaxants are used as an assistant steadies the pelvis while the surgeon applies longitudinal traction to get the femoral head under the acetabulum in order to flex and upward lift the hip. The internal rotation is corrected at this stage and a click should be felt as the hip reduces. Once reduced, the hip should be checked for stability and an X-ray should be performed in order to confirm the hip is reduced properly. The hip should have a full range of motion and be in neutral rotation. With an anterior dislocation (A) the lower limb is lengthened, the hip abducted and the foot is in external rotation. As the femur head is either anterior in the groin or in the obturator fossa it can obstruct the femoral vein causing thrombosis and possible pulmonary embolism. X- ray signs of an anterior hip dislocation are the lesser trochanter being more visible, due to external rotation. The hip is abducted and the femur head is usually inferior to the acetabulum. Shenton's line is also broken. Femoral neck (B) and hip fractures (C) are seen on X-rays as a thin like or break in the bone, although both of these fractures are common in the elderly population. One Step Further Question: Hip fractures are common in which type of bone metabolism disorder? Answer: Osteoporosi

A 24-year-old man presents with wrist pain after a fall on his outstretched right hand. Examination reveals tenderness at the base of the first metacarpal in the anatomic snuffbox and pain with axial load on the thumb. The patient's X-ray is shown above. What management is indicated? AAcetaminophen and primary care follow up BRemovable soft wrist splint for 2 weeks CSugar tong splint and orthopedics follow up DThumb spica splint and orthopedics follow upCorrect Answer

Correct Answer ( D ) Explanation: The patient has an examination concerning for an occult scaphoid fracture and requires immobilization with a thumb spica splint and follow up with an orthopedic surgeon. The scaphoid bone is the most commonly fractured carpal bone. It typically occurs after a fall on an outstretched hand (FOOSH). There are three types of fractures: 1) fractures of the tuberosity and distal pole, 2) fractures of the waist and 3) fractures of the proximal pole. Patients present with pain at the anatomic snuff box or distal radius. Physical examination reveals tenderness and swelling of the anatomic snuff box and may have increased pain with axial compression of the first metacarpal (Watson's scaphoid shift test). Additionally, pain may be increased with thumb to index finger pinch. Diagnostic testing should begin with AP, lateral and oblique plain radiographs of the wrist. A scaphoid view (X-ray with wrist in ulnar deviation) can increase the likelihood of detecting small scaphoid fractures on X-ray. Unfortunately, plain radiographs miss 15% of scaphoid fractures. Patients with missed fractures that are not immobilized are at an increased risk for fracture nonunion and long-term chronic arthritis. Therefore, splint immobilization with a thumb spica is indicated in all patients who have a clinical suspicion for scaphoid fractures regardless of the X-ray findings. Acetaminophen and primary care follow up (A) is inadequate because of the risk of a missed scaphoid fracture. A removable splint (B) does not guarantee proper immobilization and will not prevent nonunion. A sugar tong splint (C) extends from the elbow to the mid hand but does not immobilize the first metacarpal joint. One Step Further Question: How long should a scaphoid fracture be immobilized? Answer: At least 6 - 12 weeks.

Which of the following statements is true regarding this condition? see image A Associated with recent infection B Imaging can be normal C Most commonly occurs in boys ages four to eight years D Pain can be referred to the knee

Correct Answer ( D ) Explanation: The radiograph represents slipped capital femoral epiphysis (SCFE), a disorder seen in early childhood characterized by chronic slipping of the femoral epiphysis of the hip. SCFE is the most common cause of hip disability in adolescents. Clinically, the child develops hip pain referred to the thigh or knee. Internal rotation of the hip induces pain. It is more common in obese, African American males. Other risk factors include hypothyroidism, growth hormone deficiency, and renal osteodystrophy. Transient tenosynovitis (A) is a cause of pediatric limp typically preceded by a viral infection. Legg-Calve-Perthes disease (C) is an idiopathic avascular necrosis that most commonly occurs in boys aged four to eight years, whereas SCFE most commonly occurs in obese boys with an average age of 12 to 16 years. Imaging is not normal (B) and will demonstrate epiphyseal slippage. In subtle cases, MRI may be needed for confirmation, but will show abnormalities. One Step Further Question: Do girls or boys develop SCFE at a younger age? Answer: Girls typically present at a younger age (10 to 14 years) as compared to boys (12 to 16 years).

A 19-year old boy presents to the emergency room for wrist pain after he fell off of his skateboard. An X-ray is seen, see image. Which of the following is the correct diagnosis? ADistal radius fracture BFifth metacarpal fracture CLunate fracture DScaphoid fracture

Correct Answer ( D ) Explanation: The scaphoid is the most frequently fractured carpal bone and fractures of this bone are more common in males 15-30 years of age and are rarely seen in young children and infants. The primary mechanism of injury is a fall on the outstretched hand with an extended, radially deviated wrist. On physical exam, there may be mild wrist swelling or bruising and, possibly, fullness in the anatomic snuffbox, suggesting a wrist effusion. Anatomic snuffbox tenderness and tenderness of the scaphoid tubercle on palpation are classic hallmarks found on physical exam. Radiographic imaging is required to diagnose this fracture, however initially it may be difficult to see on plain films. As a basic rule, in a patient with a clinically suspected scaphoid fracture but negative initial radiographs it is reasonable to apply a short arm thumb spica splint and reevaluate the patient in two weeks. If a cast is not applied initially, the fracture can worsen over the following months. Nondisplaced distal fractures heal well with strict immobilization in a well-molded short arm thumb spica. Fractures with even small amounts of displacement are prone to nonunion, and operative treatment is recommended. Avascular necrosis of the scaphoid bone is a complication seen in nonunion fractures. Lunate fractures (C) are rare in occurrence however, similar to scaphoid fractures, they also occur with axial loading to the dorsiflexed wrist. The difference is that lunate fractures are correlated more with ulnar deviation of the wrist during impact. Anatomic snuffbox tenderness is not elicited on palpation for lunate fractures due to the anatomical location of the lunate bone on the ulnar side. Distal radius fractures (A) occur in young adults from high-impact falls and accidents where as they occur from falling on an outstretched hand They commonly occur in older adults due to the fact that older adults have weaker bone structures. Radial fractures are commonly seen in patients with osteoporosis. The median nerve is always compressed in a radial fracture and therefore median nerve function should always be assessed in this situation. Fractures of the fifth metacarpal bone (B) most commonly result from a direct blow to the ulnar border of the hand when the fist is closed, hence why the common name "boxer's fracture" is designated for this type. Localized tenderness, depression and swelling of this are common clinical presentations. One Step Further Question: Colles fracture is a fracture of which bone? Answer: Colles fracture is a distal radial fracture that occurs from falling onto an outstretched wrist.

A 21-year-old woman is brought in by ambulance from a soccer game where she was kicked by a teammate as her left leg was planted. Per the ambulance report, the patient was found with the knee bent completely under her, crying in severe pain. She was unable to bear weight on the extremity at the scene. On examination, there is no gross bony deformity of the left leg, knee, or thigh. Peri-patellar ecchymosis and a significant effusion are noted. The knee hyperextends when the leg is lifted by the heel and the knee joint is extremely unstable on valgus and varus stress. Femoral, patellar, posterior tibial, and dorsalis pedis pulses are present. Which of the following is the most likely diagnosis in this patient? A Anterior cruciate ligament tear B Medial meniscus tear C Patellar dislocation D Tibiofemoral dislocation

Correct Answer ( D ) Explanation: Tibiofemoral dislocation is a true limb-threatening emergency. It is caused by multiple ligamentous tears due to hyperextension, posterior force to the anterior tibia, or force to the femur or fibula. Tibiofemoral dislocation is most commonly caused by a motor vehicle collision, but can be caused by sports injuries, falls or even spontaneously in very obese patients. Because of the severe ligamentous damage, many tibiofemoral dislocations will spontaneously reduce prior to presentation. Knee instability in multiple directions should raise suspicion for tibiofemoral dislocation, even if no gross abnormalities are present. Hemarthrosis or significant ecchymosis may also be present on examination. Examination should include a thorough neurovascular check, include an ankle-brachial index and a motor and sensory exam. Tibiofemoral dislocations, once recognized, should be immediately manually reduced. Serial neurovascular checks should follow, as damage to neurovascular structures are common. Anterior cruciate ligament (ACL) tears (A) may occur with contact or non-contact valgus stress on the knee joint. Patients typically present after a "popping" sensation in the knee with immediate associated effusion due to hemarthrosis. While this patient likely has an associated ACL tear, a tibiofemoral dislocation is associated with multiple ligamentous tears, so this is not the best answer. Medial meniscus tears (B) typically occur due to a twisting force on a flexed knee with a planted foot. Presentation includes a gradual effusion, joint line tenderness, and palpable catching on a McMurray maneuver. The knee is typically not unstable as in the presentation above. Patellar dislocations (C) occur due to twisting of a flexed knee. The patella is commonly palpated laterally on exam, although medial dislocations are also possible. This does not cause instability of the knee and does not threatened neurovascular structures. One Step Further Question: What physical exam finding is a contraindication to manual reduction of a tibiofemoral dislocation? Answer: Dimple sign.

A 19-year old man presents to the ED after rolling his left ankle playing basketball earlier in the evening. X-rays of the ankle do not reveal a fracture. The exam reveals a positive anterior drawer test with no endpoint on the left. Which of the following is the most likely diagnosis? AAchilles' tendon rupture BGrade 1 tear of the calcaneofibular ligament CGrade 2 tear of the deltoid ligament DGrade 3 tear of the anterior talofibular ligament

D Based on the physical exam findings, a grade 3 tear of the anterior talofibular ligament (ATF) is the most likely diagnosis as the anterior drawer test is specific for the ATF and the lack of endpoint indicates a grade 3 or complete rupture of the ligament. The ATF is the most commonly injured ligament in an inversion sprain and is often the ligament that tears first. Inversion ankle sprains are classified by grade with grade 1 being the least severe and grade three being the most severe generally resulting in instability of the ankle and inability to bear weight. In addition to malleolar tenderness, the inability to bear weight if both at the time of injury and in the emergency department is an indication for X-ray via the Ottawa ankle rules

A soccer player presents to the emergency department after being kicked in the shin by his opponent an hour ago. Upon presentation he is diaphoretic and screaming in pain. You assess his lower extremity range of motion and notice that he has extreme pain with passive range of motion of his foot. He complains of numbness and has 1+ distal pulses on the affected limb. What is the best treatment for this patient? AAdmit to orthopedics for observation BIncrease pain medications CSplint the leg DSurgical fasciotomy

D Surgical fasciotomy This patient presents with acute compartment syndrome, and the best treatment for this patient is an immediate surgical fasciotomy. Compartment syndrome occurs when there is an elevated amount of pressure within a confined muscle compartment that can lead to decreased circulation and therefore tissue ischemia. Patients with compartment syndrome present with pain out of proportion to exam, pain with passive range of motion, and sometimes neurologic symptoms such as paresthesia or paralysis. Distal pulses may be diminished or absent. Acute compartment syndrome is a surgical emergency and a fasciotomy must be performed as quickly as possible. A fasciotomy is a procedure performed to open the fascia in order to release the pressure within the affected compartment. This is the only effective treatment for acute compartment syndrome. Admitting to orthopedics for observation (A) will delay time to perform fasciotomy and may lead to limb loss or death. In acute compartment syndrome, it is necessary that the patient be sent directly to the operating room to decompress the affected area. Increasing pain medications (B) will not help to treat the disorder. In patients with acute compartment syndrome, their pain is typically out of proportion to their injury, and an increase in pain medication will not help to decrease their symptoms or fix the source of their pain. Splint the leg (D) is the opposite of what should be done in cases of acute compartment syndrome. Any compressive devices, splints, casts, or other restrictive clothing that would cause any increase in pressure to the extremity should be removed.

A 51-year-old man presents with low back pain that has been worsening over the past several years. Radiographs of the spine demonstrate fusion of the sacroiliac joints and a "bamboo spine." Blood tests reveal that he is human leukocyte antigen B27 positive. Which of the following findings on physical exam would be most consistent with the suspected diagnosis? Numbness in the groin area Pain that is worse with walking and relieved with sitting Sudden onset sexual dysfunction Tenderness over the spine and sacroiliac joints

D Tenderness over the spine and sacroiliac joints Tenderness over the spine and sacroiliac joints is consistent with ankylosing spondylitis. This patient's HLA-B27 positive status and X-ray findings of sacroiliac fusion and "bamboo spine" suggest this disease process. "Bamboo spine" is characterized by flowing ligamentous ossification and syndesmophyte formation on radiographs. Ankylosing spondylitis typically presents with symptoms of low back or hip pain beginning before age 40 that may worsen over time. It is an inflammatory disease associated with the HLA-B27 genotype. Men are typically more often affected than women. Physical exam will show limited range of motion of the spine with flexion and extension and tenderness over the sacroiliac joints. Kyphosis may occur in individuals of advanced age and these individuals may progress to have a stooped posture. The most common distribution of pain is in the lower back and sacroiliac joints. Treatment includes nonsteroidal anti-inflammatory drugs and physical therapy to reduce pain. If patients fail conservative measures, tissue necrosis factor blockers are recommended. Numbness in the groin area (A) and sudden onset sexual dysfunction (C) are both symptoms of cauda equina syndrome. This may be accompanied by numbness or weakness in one or both of the legs, as well as bowel or bladder dysfunction. Pain that is worse with walking and relieved with sitting (B) is typically associated with spinal stenosis.

Question: If there is severe tenderness and swelling at the medial malleolus, which fracture must be suspected?

Maisonneuve fracture defined by fracture of the medial malleolus, disruption of the syndesmosis and fracture of the proximal fibula.

Which nerve provides innervation to the ventral surface of the thumb, index, and long finger?

Median nerve

A 25-year-old carpenter presents to your primary care clinic with acute, atraumatic low back pain. This is his first episode of back pain. He denies weakness, numbness, fever or bowel and bladder incontinence. Which of the following pain descriptors, gathered from a detailed history and examination, would most prompt you to order imaging on this initial visit? AAxial pain BFlexion-based pain CNight pain DPain began after lifting a heavy object

Night pain The evaluation of back pain begins with assessing for "red flags," such as night pain and weight loss (suspicious for tumor), fever, chills, and sweats (consider infection), acute bony tenderness (consider fracture), morning stiffness lasting > 30 minutes in young adults (consider seronegative spondyloarthropathy, and any neurologic deficit or bowel or bladder involvement (consider nerve root compromise). Acute lumbar sprain is usually the result of repeated lifting or twisting, typically occurring in young men with laborious vocation. Axial lumbar pain is typical of sprain and internal disk disruption. Radicular pain is typical of disk bulge or herniation with nerve root encroachment. Flexion-based axial lumbar pain is typical of disk pathology, whereas extension-based axial lumbar pain suggests facet joints, central stenosis or foraminal stenosis are the main pain generators. Imaging plays a limited role in acute sprain, however, it should be considered for acute lumbar pain associated with any neurologic abnormality, trauma, concerning past medical or family history, or any case involved with fever, sweats, chills, weight loss, night pain or pain at rest.

A patient presents to the Emergency Department with wrist and hand weakness. He is holding his wrist in flexion and his hand is hanging limply. He is unable to extend his wrist against resistance. Which of the following nerves is most likely injured? A Axillary nerve B Median nerve C Radial nerve D Ulnar nerve

Radial Nerve Injury! Wrist drop! Cant make thumbs up Loss of extension of thumb, wrist Correct Answer ( C ) Explanation: The patient has wrist drop, which is caused by injury to the radial nerve. The radial nerve innervates the dorsal extrinsic muscles in the forearm, which function in wrist and metacarpophalangeal (MCP) joint extension, as well as abduction and extension of the thumb. Radial nerve motor function can be assessed by having the patient extend the wrist against resistance. Patients with an intact radial nerve should be able to make the "thumbs up" sign. Sensation is tested in the first dorsal web space. Radial nerve injury has many causes, including trauma to the brachial plexus or the humerus (the radial nerve runs along the lateral border of the humerus). Additionally, prolonged use of crutches, which applies pressure along the nerve, can result in a radial nerve palsy. Similarly, "Saturday night palsy" refers to an intoxicated individual falling asleep with their arm slung over a chair, compressing the radial nerve. The axillary nerve (A) innervates the deltoid, teres minor, and the long head of the triceps. Injury to the axillary nerve leads to shoulder and elbow motor deficits. The median nerve (B) innervates the flexor muscles of the wrist, the thenar muscles, and the lumbricals to the 2nd and 3rd digits. Median nerve motor function can be assessed by testing thumb opposition and pincer grasp. Patients with an intact median nerve should be able to make the "A-OK" sign. The ulnar nerve (D) innervates the hypothenar muscles, most of the intrinsic muscles of the hand, and the adductor pollicis. Ulnar nerve motor function can be tested with thumb opposition and pincer grasp. Damage to the distal ulnar nerve can result in claw hand, in which the little and ring fingers are held in flexion at the interphalangeal joint. One Step Further Question: Which two arteries comprise the dual blood supply to the hand? Answer: Radial and ulnar arteries


Conjuntos de estudio relacionados

TORT/TORS (Latin Root): to twist

View Set

Final Exam Engine Performance week 1-5

View Set

PSY 108 7a) Basic Learning Concepts and Classical Conditioning

View Set

Liberal Challenges and Revolutions

View Set

CS165 Exam 4 StudySet: Stacks/Queues/Priority Queue

View Set